Science

NCERT | Science | Objective | Class-XII | सामान्य विज्ञान (जीव विज्ञान)

WhatsApp Group Join Now
Telegram Group Join Now

NCERT | Science | Objective | Class-XII | सामान्य विज्ञान (जीव विज्ञान)

जीव विज्ञान

1. जीवों में जनन

1. निम्नलिखित कथनों पर विचार कीजिये-
1. फंजाई जगत के सदस्य लैंगिक जननीय संरचनाओं द्वारा जनन करते हैं।
2. अलैंगिक जनन में एक जैसे जीव अथवा अभिमुख लिंग वाले जीवों द्वारा युग्मकों का निर्माण किया जाता है।
उपर्युक्त कथनों में से कौन सा/से सत्य है/हैं?
(a) केवल 1
(b) केवल 2
(c) 1 और 2 दोनों
(d) न तो 1 और न ही 2
उत्तर : (d)
व्याख्या : फंजाई जगत के सदस्य एवं साधारण पादप जैसे शैवाल विशेष अलैंगिक जननीय (Asexual Reproduction) संरचनाओं द्वारा जनन करते हैं। जबकि लैंगिक जनन (Sexual Reproduction) में एक जैसे जीव अथवा विपरीत (अभिमुख) लिंग वाले भिन्न जीवों द्वारा नर तथा मादा युग्मक (Male & female Gametes) का निर्माण शामिल है। अतः दोनों कथन असत्य हैं।
अतः विकल्प (d) सही उत्तर है।
2. आंतरिक निषेचन (Internal Fertilization) के संदर्भ में निम्नलिखित कथनों पर विचार कीजिये-
1. आंतरिक निषेचन में युग्मक संलयन जीव शरीर के भीतर संपन्न होता है।
2. बौनी फिश एवं मेंढकों में आंतरिक निषेचन होता है।
3. बीजीय पादपों में अचलनशील नर युग्मक पराग नली द्वारा मादा युग्मक तक पहुँचते हैं।
उपर्युक्त कथनों में से कौन सा/से सत्य है/हैं?
(a) केवल 1 और 2
(b) केवल 2
(c) केवल 1 और 3
(d) 1, 2 और 3
उत्तर : (c)
व्याख्या : बहुत सारे स्थलीय जीवों में जैसे- सरीसृप, पक्षी तथा स्तनधारी एवं अधिकतर पादप (ब्रायोफाइट्स, टेरिडोफाइट्स, एंजियोस्पर्म) में युग्मक संलयन जीव शरीर के भीतर संपन्न होता है। यह प्रक्रिया आंतरिक निषेचन (Internal Fertilization) कहलाती है।
• आंतरिक निषेचन मादा शरीर के अंदर होता है।
• बौनी फिश एवं मेंढकों में बाह्य निषेचन (External Fertilisation) होता है।
• बीजीय पादपों में अचलनशील नर युग्मक पराग नली द्वारा मादा युग्मक तक पहुँचते हैं।
अतः विकल्प (c) सही उत्तर है।
3. निम्नलिखित पर विचार कीजिये-
1. केंचुआ
2. फीता कृमि
3. तिलचट्टा
4. जोंक
उपर्युक्त प्राणियों में से कौन-सा/से एकलिंगी प्राणी (Unisexual Species) का/के उदाहरण है/हैं?
(a) केवल 1 और 3
(b) केवल 1, 2 और 3
(c) केवल 3
(d) केवल 2, 3 और 4
उत्तर : (c)
व्याख्या: जब नर और मादा जनन अंग अलग-अलग जीव में होते हैं. तो उसे एकलिंगी (Unisexual) कहते हैं। तिलचट्टा एकलिंगी प्राणी का उदाहरण है।
• प्राणियों में केंचुआ, फीता कृमि तथा जोंक द्विलिंगी प्राणियों के प्रारूपिक उदाहरण हैं। इसमें नर तथा मादा जनन अंग दोनों ही (उभयलिंगी/द्विलिंगी) एक प्राणी में पाए जाते हैं।
अतः विकल्प (c) सही उत्तर है।
4. जब किसी बीज का अंकुरण होता है तो उससे नवजात पौधे का निर्माण होता है। ये नवजात पौधें अपने जीवन में वृद्धि की एक निश्चित अवस्था एवं परिपक्वता तक पहुँचते हैं, इसके पश्चात् ही ये लैंगिक जनन कर सकते हैं। पादपों में यह प्रक्रिया कहलाती है-
(a) कायिक प्रावस्था
(b) पुनरुद्भवन चरण
(c) जीर्णता चरण
(d) इनमें से कोई नहीं
उत्तर : (a)
व्याख्या: सभी जीव अपने जीवन में वृद्धि की एक निश्चित अवस्था एवं परिपक्वता तक पहुँचते हैं, इसके पश्चात् ही लैंगिक जनन कर सकते हैं। वृद्धि का यह काल किशोर प्रावस्था (Juvenile Phase) कहलाता है। पादपों में यह कायिक प्रावस्था (Vegetative Phase) कहलाती है।
अतः विकल्प (a) सही उत्तर है ।
5. जीवों के लिये जनन के महत्त्व के संदर्भ में निम्नलिखित कथनों पर विचार कीजिये-
1. इससे जीवों को उत्तरजीविता में मदद मिलती है।
2. जनन जीवों के लिये अति आवश्यक जैविक प्रक्रिया है।
3. जनन द्वारा लाभदायक विभिन्नताएँ एक पीढ़ी से दूसरी पीढ़ी तक स्थानान्तरित होती हैं।
उपर्युक्त कथनों में से कौन – सा / से सत्य है/हैं?
(a) केवल 1
(b) केवल 2 और 3
(c) केवल 1 और 2
(d) 1, 2 और 3
उत्तर : (d)
व्याख्या : जनन जीवों का एक अति महत्त्वपूर्ण लक्षण है। यह एक अति आवश्यक जैविक प्रक्रिया है, जिसके द्वारा न सिर्फ जीवों को उत्तरजीविता में मदद मिलती है बल्कि इससे जीवों एवं उसकी प्रजातियों की निरंतरता बनी रहती है। अतः कथन (1) व (2) दोनों सही हैं।
• जनन एक ऐसा माध्यम है जिसके द्वारा लाभदायक विभिन्नताएँ एक पीढ़ी से दूसरी पीढ़ी तक स्थानांतरित होती हैं। अतः जनन जैव विकास में भी सहायक होता है। अतः कथन (3) सही है।
अत: विकल्प (d) सही उत्तर है।
6. कायिक प्रवर्धन (Vegetative Propagation) के संदर्भ में निम्नलिखित कथनों पर विचार कीजिये-
1. यह लैंगिक जनन का एक प्रकार होता है।
2. इसमें संतति निर्माण पौधों के कायिक भागों से होता है।
3. अदरक का प्रकंद, ब्रायोफिलम की पर्ण कलिकाएँ आदि पुष्पीय पादपों में कायिक प्रवर्धन के उदाहरण हैं।
उपर्युक्त कथनों में से कौन-सा/से कथन असत्य है/हैं?
(a) केवल 1
(b) केवल 1 और 3
(c) केवल 2 और 3
(d) 1, 2 और 3
उत्तर : (a)
व्याख्या: कायिक प्रवर्धन (Vegetative Propagation) अलैंगिक जनन का एक प्रकार होता है, जिसमें संतति निर्माण पौधों के कायिक भागों से होता है। अतः कथन 1 गलत तथा 2 सही है।
• प्राकृतिक कायिक प्रवर्धन पौधों की जड़, स्तंभ अथवा पत्तियों से हो सकता है।
• अदरक का प्रकंद, ब्रायोफिलम की पर्ण कलिकाएँ, जल हायसिंथ की भूस्तरिका आदि पुष्पीय पादपों में कायिक प्रवर्धन के उदाहरण हैं। अतः कथन 3 सही है।
अतः विकल्प (a) सही उत्तर है।

2. पुष्पी पादपों में लैंगिक प्रजनन

1. निम्नलिखित कथनों पर विचार कीजिये-
1. पुमंगों (Androecium) से भरपूर पुंकेसरों का गोला नर जनन अंग का तथा जायांग (Gynoecium) मादा जनन अंग का प्रतिनिधित्व करता है।
2. जब एक परागकोष अपरिपक्व होता है, तब सजातीय कोशिकाओं का समूह लघुबीजाणुधानी के केंद्र में स्थित होता है। उपर्युक्त कथनों में से कौन-सा/से सत्य है/हैं?
(a) केवल 1
(b) केवल 2
(c) 1 और 2 दोनों
(d) न तो 1 और न ही 2
उत्तर : (c)
व्याख्या: पुष्प में नर और मादा जनन संरचनाएँ-
• पुमंग तथा जायांग विभेदित एवं विकसित रहती हैं। पुमंगों से भरपूर पुंकेसरों का गोला (एक चक्कर) नर जनन अंग का तथा जायांग स्त्री (मादा) जनन अंग का प्रतिनिधित्व करता है।
• जब एक परागकोष अपरिपक्व होता है तब घने सुसंबद्ध सजातीय कोशिकाओं का समूह जिसे बीजाणुजन ऊतक कहते हैं, लघुबीजाणुधानी के केंद्र में स्थित होता है। अतः कथन 2 सही है।
अतः विकल्प (c) सही उत्तर है।
2. प्रत्येक कोशिका एक सक्षम पराग मातृकोशिका होती है। एक पराग मातृकोशिका से अर्द्धसूत्री विभाजन द्वारा लघुबीजाणु के निर्माण की प्रक्रिया कहलाती है-
(a) परागकोष (Anther)
(b) परागकण (Pollengrains)
(c) लघुबीजाणुधानी (Microsporangium)
(d) जननछिद्र (Germ Pores)
उत्तर : (c)
व्याख्या: प्रत्येक कोशिका एक सक्षम पराग मातृकोशिका होती है। एक पराग मातृकोशिका से अर्द्धसूत्री विभाजन द्वारा लघुबीजाणु के निर्माण की प्रक्रिया को ‘लघुबीजाणुधानी’ कहते हैं।
• एक विशिष्ट पुंकेसर दो भागों में विभक्त रहता है : इसमें लंबा एवं पतला डंठल तंतु (फिलामेंट) कहलाता है तथा अंतिम सिरा सामान्यतः द्विपालिक संरचना परागकोष कहलाता है।
• परागकण – यह नर युग्मकोद्भिद् का प्रतिनिधित्व करता है। यह सामान्यतः गोलाकार (गोलीय) होते हैं, जिनका व्यास लगभग 25.50 माइक्रोमीटर होता है। इसमें सुस्पष्ट रूप से दो पर्तो वाली भित्ति होती है।
• जननछिद्र- परागकण के बाह्यचोल में सुस्पष्ट द्वारक या रंध्र होते हैं जिन्हें जननछिद्र कहते हैं।
अतः विकल्प (c) सही उत्तर है।
3. परागकण (Pollengrains) के संदर्भ में निम्नलिखित कथनों पर विचार कीजिये –
1. परागकण की आंतरिक भित्ति सेल्यूलोज एवं पेक्टिन की बनी होती है।
2. जब परागकण परिपक्व होता है तब उसमें कायिक कोशिका (Vegetative Cell) तथा जनन कोशिकाएँ (Generative cell) समाहित होती हैं।
उपर्युक्त कथनों में से कौन-सा/से सत्य है/हैं?
(a) केवल 1
(b) केवल 2
(c) 1 और 2 दोनों
(d) न तो 1 और न ही 2
उत्तर : (c)
व्याख्या: परागकण नर युग्मकोद्भव का प्रतिनिधित्व करता है। यह सामान्यतः गोलाकार होते हैं, जिनका व्यास लगभग 25.50 माइक्रोमीटर होता है। इनमें सुस्पष्ट रूप से दो परतों वाली भित्ति होती है। कठोर बाहरी भित्ति को बाह्यचोल कहते हैं जो कि स्पोरोपोलेनिन से बनी होती है।
• परागकण की आंतरिक भित्ति को अंतः चोल कहा जाता है। यह एक पतली तथा सतत् परत होती है जो सेल्यूलोज एवं पेक्टिन की बनी होती है। जब परागकण परिपक्व होता है तब उसमें दो कायिक कोशिका तथा जनन कोशिकाएँ समाहित होती हैं। अतः दोनों कथन सत्य हैं।
अत: विकल्प (c) सही उत्तर है।
4. निम्नलिखित पर विचार कीजिये-
1. परागकण
2. परागमातृ कोशिका
3. लघुबीजाणुचतुष्क
4. बीजाणुजन ऊतक
5. नर युग्मक
उपर्युक्त को सही विकासक्रम में व्यवस्थित कीजिये-
(a) 4, 2, 1, 3, 5
(b) 4, 2, 3, 1, 5
(c) 2, 1, 4, 3, 5
(d) 2, 4, 1, 3, 5
उत्तर : (b)
व्याख्या: बीजाणुजन ऊत्तक → परागमातृ कोशिका → लघुबीजाणुचतुष्क → परागकण → नर युग्मक।
अतः विकल्प (b) सही उत्तर है।
5. निम्नलिखित पर विचार कीजिये –
1. वर्तिकाग्र ( Stigma )
2. वर्तिका ( Style)
3. अंडाशय (Ovary)
4. कायिक कोशिका (Vegetative cell)
उपर्युक्त में से कौन सा/से स्त्रीकेसर (Pistil) के भाग है/हैं?
(a) केवल 3
(b) केवल 2, 3 और 4
(c) केवल 1, 2 और 3
(d) 1, 2, 3 और 4
उत्तर : (c)
व्याख्या : जायांग (Gynoecium) पुष्प के मादा जनन अंग का प्रतिनिधित्व करता है। जायांग एक स्त्रीकेसर (एकांडपी) या बहु स्त्रीकेसर (बहुअंडपी) हो सकते हैं। जहाँ पर ये एक से अधिक होते हैं, वहाँ स्त्रीकेसर (Pistil) आपस में संगलित (युक्तांडपी) या फिर आपस में स्वतंत्र (वियुक्तांडपी) हो सकते हैं। प्रत्येक स्त्रीकेसर में तीन भाग होते है-
1. वर्तिकाग्रः परागकणों के वितरण मंच का काम करता है।
2. वर्तिका : एक दीर्घकृत पतला भाग है जो वर्तिका के नीचे होता है।
3. अंडाशयः स्त्रीकेसर के आधार पर उभरा हुआ भाग अंडाशय होता है।
अतः विकल्प (c) सही उत्तर है।
6. निम्नलिखित कथनों पर विचार कीजिये-
1. केवल कार्यशील गुरुबीजाणु मादा युग्मकोद्भिद् (Female Gametophyte) के रूप में विकसित होता है।
2. बीजांडकाय (Nucellus) केंद्रक की कोशिकाओं से आरक्षित आहार सामग्री होती है। प्रचुरता
3. एक बीजांड (Ovule) में सामान्यत: अकेला भ्रूणकोष होता है।
उपर्युक्त कथनों में से कौन – सा/से सत्य है/हैं?
(a) केवल 2
(b) केवल 1 और 2
(c) केवल 2 और 3
(d) 1, 2 और 3
उत्तर : (d)
व्याख्या : अधिकांश पुष्पी पादपों में गुरुबीजाणुओं में से एक कार्यशील होता है जबकि अन्य तीन अल्पविकसित हो जाते हैं, केवल कार्यशील गुरुबीजाणु स्त्री युग्मकोद्भिद् (Female Gametophyte) के रूप में विकसित होता है। अतः कथन 1 सही है।
• अध्यावरणी ( Integuments) से घिरा हुआ कोशिकाओं का एक पुंज होता है, जिसे बीजांडकाय (Nucellus) कहते हैं। बीजांडकाय केंद्रक की कोशिकाओं में प्रचुरता से आरक्षित आहार सामग्री होती है। अतः कथन 2 सही है।
• बीजांडकाय भ्रूणकोष या मादा युग्मकोद्भिद् में स्थित होता है। एक बीजांड में सामान्यतः एक अकेला भ्रूणकोष होता है, जो एक गुरुबीजाणु से निर्मित होता है। अतः कथन 3 सही है।
अतः विकल्प (d) सही उत्तर है।
7. निम्नलिखित पर विचार कीजिये –
1. वैलिसनेरिया (Vallisneria)
2. हाइड्रिला (Hydrilla)
3. जलकुंभी (Water Hyacinth)
4. कुमुदिनी (Water Lily)
उपर्युक्त में से किस जलीय पौधे (Aquatic Plants) का परागण कीटों एवं वायु द्वारा होता है-
(a) केवल 1
(b) केवल 1 और 3
(c) केवल 3 और 4
(d) केवल 1, 2 और 3
उत्तर : (c)
व्याख्याः सभी जलीय पौधे जल को परागण (Pollination) के लिये उपयोग में नहीं लाते हैं। अधिकांश जलीय पौधे जैसे कि जलकुंभी और कुमुदिनी पानी की सतह पर पैदा होते है; और इनका परागण कीटों एवं वायु द्वारा होता है। जबकि वैलिसनेरिया और हाइड्रिला जल परागित पादपों के उदाहरण है।
अतः विकल्प (c) सही उत्तर है।
8. द्वि-निषेचन (Double Fertilization) के संदर्भ में निम्नलिखित कथनों पर विचार कीजिये-
1. द्वि-निषेचन आवृत्तबीजी (Angiosperm) पौधों में होने वाले एक जटिल निषेचन की क्रिया है।
2. द्वि-निषेचन के फलस्वरूप भ्रूणपोष (Endosperm ) का निर्माण होता है।
उपर्युक्त कथनों में से कौन – सा / से सत्य है/हैं?
(a) केवल 1
(b) केवल 2
(c) 1 और 2 दोनों
(d) न तो 1 और न ही 2
उत्तर : (c)
व्याख्या: निषेचन की क्रिया में नरयुग्मक का अंड कोशिका से मिलकर द्विगुणित युग्मनज (Diploid Zygote) बनाना तथा द्विगुणित केंद्रक का | नरयुग्मक से संलयन द्वि-निषेचन (Double Fertilization) कहलाता है।
• द्वि-निषेचन आवृत्तबीजी (Angiosperm) पौधों में होने वाली एक जटिल निषेचन की क्रिया है।
• द्वि-निषेचन के फलस्वरूप भ्रूणपोष (Endosperm ) का निर्माण होता है। भ्रूणपोष ही भ्रूण को भोज्य पदार्थ उपलब्ध कराता है। अत: दोनों कथन सत्य हैं।
अतः विकल्प (c) सही उत्तर है।
9. भ्रूण (Embryo) के संदर्भ में निम्नलिखित कथनों पर विचार कीजिये-
1. भ्रूण विकास की प्रारंभिक अवस्था एकबीजपत्री (Monocotyledons) तथा द्विबीजपत्री (Dicotyledons) में अलग-अलग होती है।
2. एक प्ररूपी द्विबीजपत्री भ्रूण (Typical Dicotyledonous Embryo) में केवल दो बीजपत्र समाहित होते हैं।
उपर्युक्त कथनों में से कौन-सा/से सत्य है/हैं?
(a) केवल 1
(b) केवल 2
(c) 1 और 2 दोनों
(d) न तो 1 और न ही 2
उत्तर : (d)
व्याख्या : भ्रूण, भ्रूणकोष या पुटी के बीजांडद्वारी सिरे (Micropylarend पर विकसित होता है, जहाँ पर युग्मनज स्थित होता है।
• भ्रूण विकास की प्रारंभिक अवस्था एकबीजपत्री तथा द्विबीजपत्री दोनों ही में समान होती है। अतः कथन 1 असत्य है।
• एक प्ररूपी द्विबीजपत्री भ्रूण में एक भ्रूणीय अक्ष (Embryonalaxis) तथा दो बीजपत्र (Two Cotyledons) समाहित होते हैं। अतः कथन 2 भी असत्य है।
अतः विकल्प (d) सही उत्तर है।
10. निम्नलिखित पर विचार कीजिये-
1. मूंगफली
2. अमरूद
3. आम
4. सरसों
5. केला
उपर्युक्त में से कौन-से उदाहरण शुष्क फल के हैं?
(a) केवल 1 और 5
(b) केवल 1, 2, 3 और 4
(c) केवल 2, 3 और 4
(d) केवल 1 और 4
उत्तर : (d)
व्याख्या: अमरूद, आम, केला आदि गूदेदार फल हैं जबकि मूंगफली और सरसों शुष्क फल हैं।
अतः विकल्प (d) सही उत्तर है।
11. कुछ आवृत्तबीजी (Angiosperm) अपने बीज में एक से अधिक भ्रूण उत्पन्न करते हैं, इस परिघटना को कहा जाता है-
(a) असंगजनन ( Apomixis)
(b) बहुभ्रूणता (Polyembryony)
(c) परिभ्रूणपोष (Perisperm)
(d) उपर्युक्त में से कोई नहीं
उत्तर : (b)
व्याख्या: कुछ आवृत्तबीजी अपने बीज में एक से अधिक भ्रूण उत्पन्न करते हैं इस परिघटना को बहुभ्रूणता (Polyembroyony) कहा जाता है।
• पुष्पी पादपों में बिना निषेचन के ही बीज पैदा करने की प्रक्रिया असंगजनन (Apomixis) कहलाती है।
• कभी-कभी कुछ बीजों जैसे- काली मिर्च तथा चुकंदर आदि में बीजांडकाय भी शेष रह जाता है। अवशिष्ठ उपस्थित बीजांडकाय परिभ्रूणपोष (Perisperm ) होता है ।
अत: विकल्प (b) सही उत्तर है।
12. निम्नलिखित पर विचार कीजिये-
1. सेब
2. स्ट्रॉबेरी
3. अखरोट
4. केला
उपर्युक्त में से कौन-सा / से अनिषेकजनित फल (Parthenocarpic Fruits) के उदाहरण है/हैं?
(a) केवल 4
(b) केवल 1, 3 और 4
(c) केवल 2, 3 और 4
(d) केवल 2 और 4
उत्तर : (a)
व्याख्या: कुछ ऐसी प्रजातियाँ है जिनमें बिना निषेचन के फल विकसित होते हैं, ऐसे फलों को अनिषेकजनित फल (Parthenocarpic Fruits) कहते हैं। इसका एक उदाहरण केला है।
• जबकि कुछेक प्रजातियों में जैसे कि सेब, स्ट्रॉबेरी, अखरोट आदि में फल की रचना में पुष्पासन भी महत्त्वपूर्ण भागीदारी देता है। इस प्रकार के फलों को आभासी फल (False Fruits) कहते हैं। अतः विकल्प (a) सही उत्तर हैं।

3. मानव जनन

1. निम्नलिखित कथनों पर विचार कीजिये-
1. नर जर्म कोशिकाएँ (Male germ Cells) अर्द्धसूत्री विभाजन के परिणामस्वरूप शुक्राणुओं का निर्माण करती हैं।
2. सर्टोली कोशिकाएँ जर्म कोशिकाओं को पोषण प्रदान करती हैं।
उपर्युक्त कथनों में से कौन-सा/से सत्य है/हैं?
(a) केवल 1
(b) केवल 2
(c) 1 और 2 दोनों
(d) न तो 1 और न ही 2
उत्तर : (c)
व्याख्या: प्रत्येक शुक्रजनक नलिका का भीतरी भाग दो प्रकार की कोशिकाओं से स्तरित होता है, जिन्हें नर जर्म कोशिकाएँ (शुक्राणुजन) (Spermatogonia) और सर्टोली कोशिकाएँ कहते हैं।
• नर जर्म कोशिकाएँ (Male germ Cells) अर्धसूत्री विभाजन के फलस्वरूप शुक्राणुओं का निर्माण करती हैं, जबकि सर्टोली कोशिकाएँ जर्म कोशिकाओं को पोषण प्रदान करती हैं। अतः दोनों कथन सत्य हैं।
अतः विकल्प (c) सही उत्तर है।
2. निम्नलिखित में से कौन सा पुरुष में प्राथमिक यौन अंग (Primary Sexual Organ) है?
(a) वृषण ( Testes)
(b) शुक्रवाहिका (Vas Deferens)
(c) शिश्न मुंड (Glans Penis)
(d) प्रॉस्टेट (पुरस्थ) (Prostate)
उत्तर : (a)
व्याख्या: पुरुष में वृषण प्राथमिक अंग या प्रजनन अंग है ।
• वृषण टेस्टोस्टेरॉन और शुक्राणु उत्पादन के लिये उत्तरदायी होता है।
• शरीर में वृषण उदर गुहा के बाहर थैली/धानी में स्थित होते हैं जिसे वृषणकोष (Scrotum) कहते हैं। वयस्कों में प्रत्येक वृषण अंडाकार होता है, जिसकी लंबाई 4.5 सेमी. और चौड़ाई 2.3 सेमी. होती है। प्रत्येक वृषण में लगभग 250 कक्ष होते हैं।
• प्रॉस्टेट मूत्राशय के ठीक नीचे स्थित नर तंत्र का हिस्सा है। यह अखरोट के आकार का होता है और मूत्रमार्ग को घेरता है।
• शिश्न मुंड – यह नर जनन अंग का अग्र भाग है।
• शुक्रवाहिका – यह एक कुंडलित ट्यूब है जो उदर गुहा में मौजूद है।
अतः विकल्प (a) सही उत्तर है।
3. निम्नलिखित पर विचार कीजिये –
1. अंडाशय का एक युग्म (Pair of Ovaries)
2. शुक्र वाहिनी (Vas Deferens)
3. गर्भाशय ( Uterus)
4. अंडवाहिनी (Fallopian tube / Oviducts)
5. स्खलनीय वाहिनी ( Ejaculatory Duct )
उपर्युक्त में कौन सा/से मादा प्रजनन तंत्र (Female Reproductive System) का हिस्सा नहीं है/हैं?
(a) केवल 2
(b) केवल 1, 2, 4 और 5
(c) केवल 2, 3 और 5
(d) केवल 2 और 5
उत्तर : (d)
व्याख्या: मादा प्रजनन तंत्र के अंतर्गत एक जोड़ी अंडाशय (Ovary) के साथ-साथ एक जोड़ी अंडवाहिनी (Oviduct ), एक गर्भाशय ( Uterus), एक गर्भाशय ग्रीवा (Cervix) तथा एक योनि (Vagina) और बाह्य | जननेन्द्रिय (External Genitalia) शामिल होती है; जबकि शुक्र वाहिनी और स्खलनीय वाहिनी दोनों नर जनन तंत्र का हिस्सा हैं।
अत: विकल्प (d) सही उत्तर है।
4. मानव शरीर में निषेचन (Fertilization) की प्रक्रिया कहाँ होती है?
(a) गर्भाशय ( Uterus)
(b) डिंबवाहिनी नलिका (Fallopian Tube)
(c) अंडाशय (Ovary)
(d) योनि (Vagina)
उत्तर : (b)
व्याख्या : शुक्राणु के साथ एक अंडाणु के संलयन की प्रक्रिया को निषेचन (Fertilization) कहते हैं। निषेचन के दौरान शुक्राणु अंडाणु के पारदर्शी अंडावरण स्तर के संपर्क में आता है और अतिरिक्त शुक्राणुओं के प्रवेश | को रोकने हेतु उसके स्तर में बदलाव प्रेरित करता है।
• मानव शरीर में निषेचन की प्रक्रिया में डिंबवाहिनी नालिका में मानव अंडे और शुक्राणुओं का मिलन शामिल है। डिंबवाहिनी नालिका को गर्भाशय नली भी कहा जाता है। यह अंडाशय से गर्भाशय तक अंडाणुओं को पहुँचाने में मदद करती है।
अतः विकल्प (b) सही उत्तर है ।
5. निम्नलिखित पर विचार कीजिये –
1. अंडोत्सर्जन चरण (Ovulation Phase)
2. पुटक चरण ( Follicular Phase)
3. आर्तव चक्र का चरण (Phase of Menstrual Cycle)
4. पीत प्रावस्था / ल्यूटियल चरण (Luteul Phase)
उपर्युक्त अवस्थाओं/चरणों को आर्तव चक्र (Menstrual Cycle) के पूर्ण विकास क्रम में व्यवस्थित कीजिये –
(a) 2, 1, 4, 3
(b) 1, 3, 2, 4
(c) 3, 2, 1, 4
(d) 3, 1, 4, 2
उत्तर : (c)
6. आर्तव चक्र में किस हार्मोन के कम होने से आर्तव होता है?
(a) टेस्टोस्टेरॉन
(b) प्रोजेस्टेरॉन
(c) एस्ट्रोजन
(d) ऑक्सीटोसिन
उत्तर : (b)
व्याख्या: प्रोजेस्टेरॉन हॉर्मोन का कम होना आर्तव चक्र का कारण बनता है । यह एंडोमेट्रियम के अस्तर के रखरखाव के लिये आवश्यक है। इस कारण से केवल प्रोजेस्टेरॉन को गर्भावस्था हॉर्मोन भी कहा जाता है।
अतः विकल्प (b) सही उत्तर है।
7. निम्नलिखित कथनों पर विचार कीजिये-
1. अपरा (प्लेसेंटा) भ्रूण को ऑक्सीजन की पूर्ति करता है।
2. सगर्भता के उत्तरार्द्ध में एस्ट्रोजन नामक हॉर्मोन स्रावित होता है।
3. मानव में सगर्भता की औसत अवधि लगभग 7 माह होती है ।
उपर्युक्त कथनों में से कौन सा/से असत्य है/हैं?
(a) केवल 1
(b) केवल 1 और 2
(c) केवल 2 और 3
(d) 1, 2 और 3
उत्तर : (c)
व्याख्या : अपरा (प्लेसेंटा), भ्रूण को ऑक्सीजन तथा पोषण की आपूर्ति एवं कार्बन डाइऑक्साइड तथा भ्रूण द्वारा उत्पन्न उत्सर्जी (Excretory) अवशिष्ट पदार्थों को बाहर निकालने का कार्य करता है। अतः कथन 1 सत्य है। .
• सगर्भता (Pregnancy) के उत्तरार्द्ध की अवधि में अंडाशय द्वारा रिलैक्सिन नामक हॉर्मोन स्रावित होता है। अतः कथन (2) असत्य है ।
• मानव में सगर्भता की औसत अवधि – लगभग 9.5 माह होती है, जिसे गर्भावधि (जेस्टेशन पीरियड) कहते हैं। अतः कथन ( 3 ) भी असत्य है।
अतः विकल्प (c) सही उत्तर है।
8. निषेचित कोशिका को कहा जाता है-
(a) भ्रूण
(b) गर्भाशय
(c) युग्मनज
(d) अंडजनन
उत्तर : (c)
व्याख्या : निषेचित कोशिका को युग्मनज (जाइगोट) कहा जाता है। युग्मनज बार-बार विभाजित होता है और निषेचन के तुरंत बाद डिंबवाहिनी की यात्रा करता हैं।
• यौन प्रजनन में नर और मादा युग्मक का निर्माण होता है। अतः विकल्प (c) सही उत्तर है।
9. निम्नलिखित पर विचार कीजिये-
1. एस्ट्रोजन
2. प्रोजेस्टेरॉन
3. ऑक्सीटोसिन
4. प्रोलैक्टिन
उपर्युक्त में से कौन-सा/से स्तन्यस्रावण हॉर्मोन है/हैं?
(a) केवल 1 और 4
(b) केवल 3
(c) केवल 2, 3 और 4
(d) केवल 1, 3 और 4
उत्तर : (b)
व्याख्या: मादा की स्तन ग्रंथियों में सगर्भता (Pregnancy) के दौरान कई प्रकार के बदलाव आते हैं और सगर्भता के अंत तक इनसे दुग्ध स्रावित होने लगता है। इस प्रक्रिया को दुग्धस्रावण (लैक्टेशन) कहते हैं।
• ऑक्सीटोसिन, स्तन्यस्रावण हॉर्मोन है।
• प्रोलैक्टिन पीयूष ग्रंथि द्वारा निर्मित एक हॉर्मोन है जो मस्तिष्क के तल पर स्रावित होता है। प्रोलैक्टिन स्तनों को बढ़ने और विकसित करने का कारण बनता है और बच्चे के जन्म के बाद दुग्ध स्रावण का कारण बनता है।
• एस्ट्रोजन – यह प्राथमिक मादा सेक्स हॉर्मोन है। यह विकास के लिये जिम्मेदार है।
• प्रोजेस्टेरॉन – यह ओव्यूलेशन के बाद गर्भावस्था की क्षमता के लिये एंडोमेट्रियम तैयार करता है।
अतः विकल्प (b) सही उत्तर है।
10. निम्नलिखित कथनों पर विचार कीजिये-
1. दुग्धस्रावण के आरंभिक कुछ दिनों तक जो दूध निकलता है, उसे कोलोस्ट्रम कहते हैं।
2. स्टेम सेल थेरेपी द्वारा डिमेंशिया, ऑटिज़म और सेरेब्रल पालसी जैसी बीमारियों का इलाज संभव है।
3. स्त्रियों में गर्भावस्था के तीसरे माह के दौरान गर्भ की पहली गतिशीलता को देखा जा सकता है।
उपर्युक्त कथनों में से कौन सा/से सत्य है/हैं?
(a) केवल 2
(b) केवल 1 और 2
(c) केवल 2 और 3
(d) 1, 2 और 3
उत्तर : (b)
व्याख्या : दुग्धस्रावण के आरंभिक कुछ दिनों तक जो दूध निकलता है उसे प्रथम स्तन्य या खीस (कोलोस्ट्रम) कहते हैं। इसमें कई प्रकार के प्रतिरक्षी (Antibody) तत्त्व समाहित होते हैं, जो नवजात शिशु में प्रतिरोधी क्षमता उत्पन्न करने के लिये आवश्यक होते हैं। अत: कथन (1) सत्य है ।
• डिमेंशिया, ऑटिज़म और सेरेब्रल पालसी जैसी बीमारियों का इलाज अब स्टेम सेल थेरेपी से संभव है। अतः कथन (2) सत्य है।
• स्त्रियों में गर्भावस्था के पाँचवे माह के दौरान गर्भ की पहली गतिशीलता और सिर पर बालों का उगना सामान्यतः देखा जा सकता है। इसलिये कथन (3) असत्य है।
अतः विकल्प (b) सही उत्तर है।
11. एक कोशिका की दो समान अनुजात कोशिकाओं में विभाजित होने की क्षमता कहलाती है-
(a) खंडन
(b) बहुखंडन
(c) निषेचन
(d) द्वि-विभाजन
उत्तर : (d)
व्याख्या : एक कोशिका की दो समान अनुजात कोशिकाओं में विभाजित होने की क्षमता को ‘द्वि-विभाजन’ कहा जाता है।
• बहुखंडन अलैंगिक प्रजनन प्रक्रिया है, जिसमें दो अनुजात कोशिकाओं के स्थान पर कई अनुजात कोशिकाओं को मूल कोशिका से उत्पन्न किया जाता है।
• बहुकोशिकीय खंडन एक प्रकार का अलैंगिक प्रजनन है, जिसमें एक जीव को टुकड़ों में विभाजित किया जाता है।
• शुक्राणु के साथ एक अंडाणु के संलयन की प्रक्रिया को निषेचन कहा जाता है।
अतः विकल्प (d) सही उत्तर है।
12. अंडजनन (Oogenesis) के संदर्भ में निम्नलिखित कथनों पर विचार कीजिये-
1. एक परिपक्व मादा युग्मक के निर्माण की प्रक्रिया को अंडजनन कहते हैं।
2. अंडजनन की शुरुआत भ्रूणीय परिवर्धन चरण के दौरान होती है।
उपर्युक्त कथनों में से कौन सा/से सत्य है/हैं?
(a) केवल 1
(b) केवल 2
(c) 1 और 2 दोनों
(d) न तो 1 और न ही 2
उत्तर : (c)
व्याख्या : एक परिपक्व मादा युग्मक के निर्माण की प्रक्रिया को अंडजनन (Oogenesis) कहते हैं, जो पुरुष के शुक्राणुजनन से स्पष्ट रूप से भिन्न है । अतः कथन ( 1 ) सत्य है।
• अंडजनन की शुरुआत भ्रूणीय परिवर्धन चरण के दौरान होती है जब कई मिलियन मातृ युग्मक कोशिकाएँ यानी अंडजननी प्रत्येक भ्रूणीय अंडाशय के अंदर विनिर्मित होती है। जन्म के बाद अंडजननी का निर्माण और उसकी वृद्धि नहीं होती है। अतः कथन (2) भी सत्य है।
अतः विकल्प (c) सही उत्तर है ।
13. निम्नलिखित में से किसमें / किनमें शुक्राणु अस्थायी रूप से रखे जाते हैं?
1. अधिवृषण
2. शुक्रवाहिका
3. मूत्राशय
4. प्रॉस्टेट
नीचे दिये गए कूट से सही उत्तर चुनिये-
(a) केवल 1
(b) केवल 1 और 2
(c) केवल 2, 3 और 4
(d) केवल 2 और 4
उत्तर : (a)
व्याख्या : शुक्राणु को अस्थायी रूप अधिवृषण में रखा जाता है। प्रत्येक शुक्राणु कोशिका को बनने 65.75 दिन लगते हैं।
• शीर्ष पर और प्रत्येक वृषण के पीछे अधिवृषण होता है, जो शुक्राणु को संग्रहित करता है। शुक्राणु कोशिकाएँ अधिवृषण में परिपक्वता प्राप्त करती है।
अतः विकल्प (a) सही उत्तर है।

4. जनन स्वास्थ्य

1. निम्नलिखित कथनों पर विचार कीजिये-
1. जनन स्वास्थ्य का अर्थ जनन के सभी पहलुओं सहित एक संपूर्ण स्वास्थ्य से है।
2. विश्व में भारत पहला देश है, जिसने राष्ट्रीय स्तर पर जनन स्वास्थ्य को लक्ष्य के रूप में प्राप्त करने के लिये कार्ययोजना बनाई।
उपर्युक्त कथनों में से कौन सा/से सत्य है/हैं?
(a) केवल 1
(b) केवल 2
(c) 1 और 2 दोनों
(d) न तो 1 और न ही 2
उत्तर : (c)
व्याख्या : विश्व स्वास्थ्य संगठन के अनुसार जनन स्वास्थ्य का अर्थ जनन के सभी पहलुओं सहित एक संपूर्ण स्वास्थ्य अर्थात् शारीरिक, भावनात्मक, व्यवहारात्मक तथा सामाजिक स्वास्थ्य है । इसीलिये कथन (1) सही है।
• विश्व में भारत पहला ऐसा देश है, जिसने राष्ट्रीय स्तर पर संपूर्ण जनन स्वास्थ्य को एक लक्ष्य के रूप में प्राप्त करने के लिये राष्ट्रीय कार्ययोजना और कार्यक्रमों की शुरुआत की। इन कार्यक्रमों को ‘परिवार नियोजन’ के नाम से जाना जाता है और इनकी शुरुआत 1951 में हुई थी। अतः कथन ( 2 ) सही है।
अतः विकल्प (c) सही उत्तर है ।
2. निम्नलिखित कारणों पर विचार कीजिये-
1. जनन योग्य व्यक्तियों की संख्या में वृद्धि होना ।
2. वैज्ञानिक एवं तकनीकी प्रगति के कारण जीवन स्तर में सुधार होना।
3. स्वास्थ्य सुविधाओं के कारण शिशु मृत्युदर एवं मातृ मृत्युदर में कमी आना।
4. सामुदायिक स्वास्थ्य कार्यक्रमों द्वारा अनेक महामारियों का समूल रूप से निवारण होना ।
5. यौन संचारित रोगों की शीघ्र पहचान व उनका समुचित उपचार करना।
6. गर्भपात, गर्भनिरोधक, आर्तव चक्र, बाँझपन संबंधी समस्याओं की पहचान करना ।
उपर्युक्त में से कौन-से कारण जनसंख्या वृद्धि के लिये उत्तरदायी हैं?
(a) केवल 2, 3 और 5
(b) केवल 1, 3 और 4
(c) केवल 1, 2, 3 और 4
(d) उपर्युक्त सभी
उत्तर : (c)
व्याख्या: किसी भी क्षेत्र में एक निश्चित समय में बढ़ी हुई आबादी या जनसंख्या को जनसंख्या वृद्धि कहते हैं। जनसंख्या वृद्धि के निम्नलिखित कारण हैं-
• स्वास्थ्य सुविधाओं के कारण शिशु मृत्युदर (Infant Mortality Rate – IMR) एवं मातृ मृत्युदर (Maternal Mortality Rate MMR) में कमी आई है।
• जनन योग्य व्यक्तियों की संख्या में वृद्धि होना ।
• अच्छी स्वास्थ्य सेवाओं के कारण जीवन स्तर में सुधार होना ।
• अशिक्षा के कारण व्यक्तियों को परिवार नियोजन के साधनों का ज्ञान न होना और परिवार नियोजन के तरीकों को पूर्ण रूप से न अपनाया जाना
• वैज्ञानिक एवं तकनीकी प्रगति के कारण खाद्यान्नों के उत्पादन में वृद्धि
• सामुदायिक स्वास्थ्य कार्यक्रमों द्वारा अनेक महामारियों का समूल निवारण होना ।
अतः विकल्प (c) सही उत्तर है ।
3. उल्बवेधन (एमीनोसैंटेसिस) के संदर्भ में निम्नलिखित कथनों पर विचार कीजिये –
1. उल्वबेधन एक घातक लिंग निर्धारण (जाँच) प्रक्रिया है।
2. इस प्रक्रिया की भारत में वैध अनुमति है।
3. इस तकनीक में माता के गर्भ से एमनीओटिक द्रव बाहर निकाला जाता है।
उपर्युक्त कथनों में से कौन-सा / से सही नहीं है/हैं?
(a) केवल 3
(b) केवल 2 और 3
(c) केवल 2
(d) 1, 2 और 3
उत्तर : (c)
व्याख्या: उल्बवेधन एक घातक लिंग निर्धारण (जाँच) प्रक्रिया है, जो हमारे देश में निषेधित है। अतः कथन ( 2 ) सही नहीं है |
• उल्वबेधन एक ऐसी तकनीक है, जिसके अंतर्गत माता के गर्भ से एमनीओटिक द्रव्य (Amniotic Fluid) का कुछ भाग बाहर निकाला जाता है। इस द्रव्य में फीट्स की कोशिकाएँ होती है जिसके गुणसूत्रों का विश्लेषण करके भ्रूण की लिंग जाँच आनुवंशिक संरचना, आनुवंशिक विकार व उपायचयी विकारों का पता लगाया जा सकता है। अतः इस जाँच प्रक्रिया का प्रमुख उद्देश्य होने वाली संतान में किसी भी संभावित दिव्यांगता अथवा विकार का पता लगाना है, जिससे आवश्यकता होने पर माता को गर्भपात कराने का आधार मिल सके। अतः कथन (1) और (3) सही हैं।
अत: विकल्प (c) सही उत्तर है।
4. निम्नलिखित पर विचार कीजिये-
1. परखनली शिशु (Test Tube Baby)
2. युग्मक अन्त: फैलोपियन स्थानांतरण (Gamete-Intra Fallopian Transfer)
3. कृत्रिम गर्भाधान (Artificial Insemination)
4. अन्तः कोशिकाद्रव्यीय शुक्राणु बेधन ( Intra – Cytoplasmic Sperm Injection)
उपर्युक्त में से कौन-सी विधियाँ बंध्य दंपतियों को संतान पाने हेतु सहायक हैं?
(a) केवल 1 और 3
(b) केवल 1, 2 और 3
(c) केवल 2, 3 और 4
(d) उपर्युक्त सभी
उत्तर : (d)
व्याख्याः बन्ध्य दंपतियों को संतान पाने हेतु सहायता देने के लिये निम्नलिखित विधियाँ हैं-
1. परखनली शिशु (Test Tube Baby)
2. युग्मक अन्तः फैलोपियन स्थानांतरण ( Gamete-Intra Fallopian Transfer)
3. अन्तः कोशिकाद्रव्यीय शुक्राणु बेधन ( Intra-Cytoplasmic Sperm Injection)
4. कृत्रिम गर्भाधान ( Artificial Insemination) अत: विकल्प (d) सही उत्तर है।
5. निम्नलिखित कथनों पर विचार कीजिये-
1. एक प्राकृतिक गर्भ निरोधक उपाय के रूप में शिशु को पूर्ण रूप से स्तनपान कराना सहायक होता है।
2. गर्भ निरोध के शल्य क्रियात्मक उपाय युग्मक परिवहन अथवा युग्मक संचार को रोकते हैं।
उपर्युक्त कथनों में से कौन – सा /से कथन सत्य है/हैं?
(a) केवल 1
(b) केवल 2
(c) 1 और 2 दोनों
(d) न तो 1 और न ही 2
उत्तर : (c)
व्याख्या: प्रसव के उपरांत शिशु को भरपूर स्तनपान कराने से अण्डोत्सर्ग नहीं होता है। अतः आर्तव चक्र के प्रारंभ न होने से गर्भ ठहरने की संभावना भी नहीं रहती है। किंतु यह प्रसव के पश्चात् लगभग 4.6 महीने तक ही प्रभावी होता है। अतः कथन (1) सत्य है।
• गर्भ निरोध के शल्य क्रियात्मक उपाय युग्मक परिवहन अथवा युग्मक संचार को रोकते हैं। कथन (2) भी सत्य है।
अतः विकल्प (c) सही उत्तर है।
6. निम्नलिखित पर विचार कीजिये –
1. खसरा
2. टी.बि.
3. गोनोरिया
4. टायफॉइड
उपर्युक्त में से कौन – सा / से यौन संचारित रोग है/हैं?
(a) केवल 3
(b) केवल 2 और 3
(c) केवल 1, 2 और 4
(d) उपर्युक्त में से कोई नहीं
उत्तर : (a)
व्याख्या : यौन संचारित रोग वे रोग हैं, जो सूक्ष्मजीवों (जीवाणु, कवक, परजीवी, विषाणु) के कारण होते हैं और मुख्य रूप से संभोग (sex) के | दौरान रक्त, वीर्य और योनि तरल पदार्थ द्वारा प्रेषित होते हैं। उदाहरणक्लैमिडिया, गोनोरिया (सुजाक), हेपेटाइटिस बी, हेपेटाइटिस सी, एचआईवी तथा सिफलिस आदि ।
अतः विकल्प (a) सही उत्तर है ।
7. पात्रे निषेचन (In Vitro Fertilization) के संदर्भ में निम्नलिखित कथनों पर विचार कीजिये-
1. इसमें प्रयोगशाला में पत्नी का या दाता स्त्री के अंडे से पति अथवा दाता पुरुष से प्राप्त शुक्राणुओं को प्रयोगशाला में युग्मनज बनने के लिये प्रेरित किया जाता है।
2. इस प्रक्रिया में निषेचित अंडे को मादा के गर्भाशय में प्रवेश कराया जाता हे।
3. अधिकांश देशों में इसके विनियमन के लिये कानून नहीं बनाए गए हैं।
उपर्युक्त कथनों में से कौन-सा / से सत्य है/हैं?
(a) केवल 1
(b) केवल 1 और 3
(c) केवल 1 और 2
(d) 1, 2 और 3
उत्तर : (c)
व्याख्या : IVF निषेचन की एक कृत्रिम प्रक्रिया है, जिसमें किसी मादा के अंडाशय से अंडे निकालकर उसका संपर्क द्रव्य माध्यम में शुक्राणुओं से कराया जाता है।
• इस प्रकार का निषेचन शरीर के बाहर किसी अन्य पात्र में कराया जाता है। इसके बाद निषेचित अंडे को मादा के गर्भाशय में प्रवेश कराया जाता है। अतः कथन (1) और (2) सत्य हैं।
• अधिकांश देशों में IVF के विनियम के लिये कानून बनाए गए हैं। इन कानूनों के तहत IVF के लिये अधिकतम आयु सीमा 40 से 50 वर्ष के बीच निर्धारित की गई है।
• उदाहरण- अमेरिका में IVF के लिये अधिकतम आयु सीमा 50 वर्ष और डिंब दान (Ovum Donation) के लिये 45 वर्ष है, वहीं ऑस्ट्रेलिया में रजोनिवृत्ति की आयु (52 वर्ष) के बाद IVF प्रतिबंधित हैं। अतः कथन (3) असत्य है।
अतः विकल्प (c) सही उत्तर है।
8. निम्नलिखित कथनों पर विचार कीजिये-
1. बंध्यकरण प्रक्रिया को पुरुषों के लिये शुक्रवाहक- उच्छेदन (वासैक्टोमी) कहा जाता है।
2. कृत्रिम गर्भाधान का प्रयोग उन पुरुषों पर किया जाता है, जिनमें शुक्राणुओं की कमी होती है।
उपर्युक्त कथनों में से कौन – सा /से सही है/हैं?
(a) केवल 1
(b) केवल 2
(c) 1 और 2 दोनों
(d) न तो 1 और न ही 2
उत्तर : (c)
व्याख्या : बंध्यकरण प्रक्रिया को पुरुषों के लिये ‘शुक्रवाहक – उच्छेदन (वासैक्टोमी) ‘ तथा महिलाओं के लिये ‘डिंबवाहिनी नलिका उच्छेदन (टूबेक्टोमी)’ कहा जाता हैं। अतः कथन 1 सही है ।
• कृत्रिम गर्भाधान (Artificial Insemination): इसका प्रयोग उन पुरुषों पर किया जाता है, जिनमें शुक्राणुओं की कमी होती है। इस विधि में पुरुष के वीर्य को एकत्रित करके स्त्री की योनि में स्थापित कर दिया जाता है। अतः कथन ( 2 ) भी सही है ।
अतः विकल्प (c) सही उत्तर है।
9. निम्नलिखित पर विचार कीजिये-
1. एलिया
2. PCR
3. VDRL
4. DNA हाइब्रिडाइजेशन
उपर्युक्त में से सिफलिस के लिये सबसे उपयुक्त जाँच है-
(a) केवल 1
(b) केवल 3
(c) केवल 2 और 4
(d) केवल 1, 3 और 4
उत्तर : (b)
व्याख्या : सिफलिस एक जीवाणु संक्रमण रोग है, जो आमतौर पर किसी संक्रमित व्यक्ति के साथ यौन संबंध बनाने पर फैलता है।
• इसके लिये सबसे उपयुक्त जाँच है- VDRL : (Venereal Disease Research Laboratory Test)
• इस जाँच से मरीज में एंटीबॉडी का पता लगाया जाता है।
अतः विकल्प (b) सही उत्तर है।

5. वंशागति तथा विविधता के सिद्धांत

1. वंशागति (Inheritance) के संदर्भ में निम्नलिखित कथनों पर विचार कीजिये-
1. वंशागति आनुवंशिकी का एक प्रकार है।
2. वंशागति वह प्रक्रम है, जिससे लक्षण जनक से संतति में जाते हैं।
उपर्युक्त कथनों में से कौन सा/से सत्य है/हैं?
(a) केवल 1
(b) केवल 2
(c) 1 और 2 दोनों
(d) न तो 1 और न ही 2
उत्तर : (b)
व्याख्या : वंशागति आनुवंशिकी का आधार है। वंशागति वह प्रक्रम है जिससे लक्षण जनक से संतति में जाते हैं तथा विविधता जनक और संतति के लक्षणों की असमानता की अवस्था है।
• इस प्रकार, जनकों या माता-पिता से संतानों में गुण – लक्षणों का अभिगमन ही वंशागति कहलाता है। अतः केवल कथन 2 सत्य है।
अतः विकल्प (b) सही उत्तर है ।
2. निम्नलिखित कथनों पर विचार कीजिये-
1. समयुग्मजी (Homozygous) में दो कारकों के दोनों युग्म विकल्पी समान होते हैं।
2. विषमयुग्मजी (Heterozygous) में दो कारकों के दोनों युग्म विकल्पी असमान होते हैं।
उपर्युक्त कथनों में से कौन सा/से सत्य है/हैं?
(a) केवल 1
(b) केवल 2
(c) 1 और 2 दोनों
(d) न तो 1 और न ही 2
उत्तर : (d)
व्याख्या: समयुग्मजी में एक कारक के दोनों युग्म विकल्पी समान होते हैं। जैसे – TT, tt । इसमें जनन के समय एक ही प्रकार के युग्मक बनते हैं। इनकी संतान जनक के समलक्षणी होती है। अतः कथन 1 असत्य है।
• विषमयुग्मजी में एक कारक के दोनों विकल्पी असमान होते हैं। जैसे- Tt। इसमें अलग-अलग प्रकार के युग्मक बनते हैं। इनकी संतान में प्रभावी व अप्रभावी दोनों लक्षण होते हैं। अतः कथन (2) भी असत्य है।
अत: विकल्प (d) सही उत्तर है।
3. प्रभाविता के नियम (Law of Dominance) के संदर्भ में निम्नलिखित कथनों पर विचार कीजिये-
1. इसमें लक्षण कारक के रूप में होते हैं।
2. यदि कारक जोड़ों के दो सदस्य असमान हों तो इनमें से एक कारक दूसरे कारक पर प्रभावी हो जाता है।
उपर्युक्त कथनों में से कौन-सा/से सत्य है/हैं?
(a) केवल 1
(b) केवल 2
(c) 1 और 2 दोनों
(d) न तो 1 और न ही 2
उत्तर : (c)
व्याख्या : प्रभाविता के नियमानुसार- लक्षणों का निर्धारण कारक नामक विविक्त (Discrete) इकाइयों द्वारा होता है। अतः कथन ( 1 ) सही है ।
• कारक जोड़ों में होते हैं। यदि कारक जोड़ों के दो सदस्य असमान हों तो इनमें से एक कारक दूसरे कारक पर प्रभावी हो जाता है। अतः कथन (2) सही है।
• F1 में केवल एक जनक लक्षण का प्रकट होना तथा F2 में दोनों जनक लक्षणों का प्रकट होना; प्रभाविता के नियम के द्वारा समझा जा सकता है। अतः विकल्प (c) सही उत्तर है।
4. निम्नलिखित पर विचार कीजिये-
1. मेंडल विश्लेषण
2. पैनेट विश्लेषण
3. वंशावली विश्लेषण
4. वाल्टर विश्लेषण
उपर्युक्त में से किसी परिवार की अनेक पीढ़ियों के लक्षणों का विश्लेषण कहलाता है-
(a) केवल 1 और 4
(b) केवल 2 और 3
(c) केवल 3
(d) केवल 4
उत्तर : (c)
व्याख्या : किसी परिवार की अनेक पीढ़ियों के लक्षणों का विश्लेषण – वंशावली विश्लेषण (Pedigree Analysis) कहलाता है।
• मानव आनुवंशिकी में वंशावली अध्ययन महत्त्वपूर्ण उपकरण होता है, जिसका उपयोग विशेष लक्षण, असामान्यता (Abnormality) या रोग का पता लगाने में किया जाता है। इस प्रक्रिया मे वंश वृक्ष (Family Tree) में एक विशेष लक्षण का पीढ़ी दर पीढ़ी विश्लेषण किया जाता है।
अतः विकल्प (c) सही उत्तर है।
5. निम्नलिखित पर विचार कीजिये –
1. जीन (Gene)
2. फीनोटाइप (Phenotype)
3. ऐलील (Allele)
4. जीनोटाइप (Genotype)
उपर्युक्त में से मेंडल के प्रयोगों में विपरीत लक्षणों की जोड़ी को क्या कहा जाता है?
(a) केवल 1
(b) केवल 2 और 4
(c) केवल 3
(d) केवल 4
उत्तर : (c)
व्याख्या : मेंडल के प्रयोगों में विपरीत लक्षणों की जोड़ी को ऐलील (Allele) कहा जाता है।
• जीनोटाइप डीएनए में जीव का एक समूह है, जो अद्वितीय गुण या विशेषताओं लिये जिम्मेदार है।
• फीनोटाइप किसी जीव की शारीरिक बनावट या विशेषता है। अतः विकल्प (c) सही उत्तर है।
6. निम्नलिखित कथनों पर विचार कीजिये-
1. एकल जीन अनेक दृश्य प्रारूप ( Phenotype) लक्षणों को प्रकट कर सकता है, ऐसे जीन को बहुप्रभावी जीन (Pleiotropic Gene) कहते हैं।
2. मनुष्य में होने वाली फेनिलकीटोमेह व्याधि (phenylketonuria) आनुवंशिक विकार है।
उपर्युक्त कथनों में से कौन-सा/से असत्य है/हैं?
(a) केवल 1
(b) केवल 2
(c) 1 और 2 दोनों
(d) न तो 1 और न ही 2
उत्तर : (b)
व्याख्या: ऐसे दृष्टांत जहाँ एकल जीन अनेक दृश्य प्रारूप (फीनोटाइप) लक्षणों को प्रकट कर सकता है, ऐसे जीन को ‘बहुप्रभावी जीन’ कहते हैं। अतः कथन (1) सत्य है।
• मनुष्य में होने वाली फेनिलकीटोन्यूरिया (Phenylketonuria) बहुप्रभाविता का उदाहरण है। यह रोग फेनिल-ऐलेनीन हाइड्रॉक्सीलेज नामक एंजाइम के लिये उत्तरदायी जीन में उत्परिवर्तन के कारण होता है। अतः कथन (2) असत्य है।
अतः विकल्प (b) सही उत्तर है।
7. निम्नलिखित कथनों पर विचार कीजिये-
1. उत्परिवर्तन (Mutation) वह क्रिया है, जो डीएनए अनुक्रम में बदलाव ला सकती है।
2. पराबैंगनी किरणें जीवों में उत्परिवर्तन पैदा कर सकती हैं।
3. सिकल सेल एनीमिया, बिंदु उत्परिवर्तन ( Point Mutation ) के उदाहरण हैं।
उपर्युक्त कथनों में से कौन-सा/से सत्य है/हैं?
(a) केवल 1
(b) केवल 3
(c) केवल 2 और 3
(d) 1, 2 और 3
उत्तर : (d)
व्याख्या : उत्परिवर्तन वह क्रिया है जो डीएनए अनुक्रम में बदलाव ला सकती है, इसके परिणामस्वरूप जीव के जीनोटाइप और फीनोटाइप में परिवर्तन आ जाता है। अतः कथन (1) सत्य है।
• उत्परिवर्तनों का जन्म अनेक रासायनिक और भौतिक कारकों द्वारा होता है। इन्हें ‘उत्परिवर्तजन’ (Mutagen) नाम दिया गया है। पराबैंगनी विकिरण, जीवों में उत्परिवर्तन पैदा कर सकते हैं। ये उत्परिवर्तजन ही हैं। अतः कथन (2) भी सत्य है।
• डीएनए के एकल क्षार युग्म के परिवर्तन भी उत्परिवर्तन को जन्म देते हैं। इसे बिंदु उत्परिवर्तन ( Point Mutation) कहते हैं। सिकल सेल एनीमिया इसका जाना माना उदाहरण है। इसलिये कथन (3) भी सत्य है।
अतः विकल्प (d) सही उत्तर है।
8. निम्नलिखित पर विचार कीजिये –
1. वर्णांधता (Colour Blindness)
2. थैलेसीमिया (Thalassemia)
3. सिकल सेल एनीमिया (Sickle Cell Anemia)
4. एड्स (AIDS)
उपर्युक्त में से कौन-सा / से आनुवंशिक विकार (Genetic disorder) है/हैं?
(a) केवल 1
(b) केवल 2 और 3
(c) केवल 2, 3 और 4
(d) केवल 1, 2 और 3
उत्तर : (d)
व्याख्या: वर्णांधता, थैलेसीमिया और सिकल सेल एनीमिया आनुवंशिक विकार संबंधित रोग (Genetic Disorder Related Disease) उदाहरण हैं।
अतः विकल्प (d) सही उत्तर है।
9. मेंडल ने प्रस्तावित किया-
(a) सहलग्नता का नियम
(b) आनुवंशिकी का नियम
(c) उपयोगिता ह्रास का नियम
(d) अर्जित गुणों की वंशागति
उत्तर : (b)
व्याख्या : मेंडल के वंशागति के नियम को ‘आनुवंशिकता का नियम’ कहते हैं। इस सिद्धांत के अनुसार एक पीढ़ी से दूसरी पीढ़ी के लक्षणों का स्थानांतरण होता है।
• मेंडल ने ये प्रयोग मटर के पौधे पर किया था। उन्होंने सात जोड़ी ऐसे लक्षणों को लेकर यह प्रयोग किया और नई पीढ़ियाँ प्राप्त की।
• मेंडल को हम ‘आनुवंशिकता का जनक’ भी कहते हैं।
अतः विकल्प (b) सही उत्तर है।

6. वंशागति के आणविक आधार

1. निम्नलिखित कथनों पर विचार कीजिये-
1. साइटोसीन एवं थायमीन डीएनए और आरएनए दोनों में मिलता है।
2. आरएनए में यूरेसिल मिलता है।
उपर्युक्त कथनों में से कौन सा/से सत्य है/हैं?
(a) केवल 1
(b) केवल 2
(c) 1 और 2 दोनों
(d) न तो 1 और न ही 2
उत्तर : (b)
व्याख्या : साइटोसीन डीएनए व आरएनए दोनों में मिलता है जबकि थायमीन डीएनए में मिलता है। अतः कथन (1) असत्य है।
आरएनए में थायमीन के स्थान पर यूरेसिल मिलता है। अतः कथन ( 2 ) सत्य है।
अतः विकल्प (b) सही उत्तर है।
2. निम्नलिखित में से किसने द्विकुंडलित डीएनए की संरचना का मॉडल प्रस्तुत किया?
(a) मेंडल
(b) लैमार्क
(c) वाटसन एवं क्रिक
(d) रदरफोर्ड
उत्तर : (c)
व्याख्या :
• मौरिस विल्किंस व रोजलिंड फ्रैंकलिन द्वारा दिये गए एक्स-रे निवर्तन आँकड़े के आधार पर 1953 में जेम्स वाटसन व फ्रांसिस क्रिक ने डीएनए की संरचना का द्विकुंडलित नमूना प्रस्तुत किया ।
अतः विकल्प (c) सही उत्तर है ।
3. यदि एक द्विरज्जुक डीएनए में 30% साइटोसीन है तो डीएनए में मिलने वाले एडिनीन की प्रतिशतता होगी-
(a) 10%
(b) 20%
(c) 60%
(d) इनमें से कोई नहीं
उत्तर : (b) व्याख्या : डीएनए में,
4. निम्नलिखित कथनों पर विचार कीजिये-
1. केंद्रक में मिलने वाली एक संरचना जिस पर न्यूक्लियोसोम मिलते हैं, उसे ‘क्रोमेटीन’ कहते हैं।
2. एक प्रारूपी केंद्रक में कुछ जगहों पर क्रोमेटीन ढीले-ढीले बँधे होते हैं, इसे ‘यूक्रोमेटीन’ कहा जाता है।
उपर्युक्त कथनों में से कौन-सा/से सही है/हैं?
(a) केवल 1
(b) केवल 2
(c) 1 और 2 दोनों
(d) न तो 1 और न ही 2
उत्तर : (c)
व्याख्या: केंद्रक में मिलने वाली एक संरचना जिस पर न्यूक्लियोसोम मिलते हैं, उसे ‘क्रोमेटीन’ कहते है। क्रोमेटीन, जालिका या ऐक्रोमेटीन (Achromatin) के बने होते हैं। ये अम्लीय रंजकों से गहरे तथा क्षारीय रंजकों से हल्के अभिरंजित होते हैं। अतः कथन (1) सही है।
• क्रोमेटीन दो प्रकार के होते हैं-
1. यूक्रोमेटीन : एक प्रारूपी केंद्रक में कुछ जगहों पर क्रोमेटीन । ढीले-ढीले (हल्के अभिरंजित) होते हैं जिसे यूक्रोमेटीन कहते है । अतः कथन (2) भी सही है।
2. हेटरोक्रोमेटीन: क्रोमेटीन जो काफी अच्छे ढंग से बँधे होते हैं व गाढ़े रंग के दिखाई पड़ते हैं, उसे ‘हेटरोक्रोमेटीन’ कहते हैं।
अतः विकल्प (c) सही उत्तर है।
5. डीएनए की एक रज्जुक से आनुवंशिक सूचनाओं का आरएनए में प्रतिलिपिकरण करने की प्रक्रिया कहलाती है-
(a) अनुलेखन (Transcription )
(b) प्रतिकृति (Replication)
(c) अव्यक्तेक (Intron)
(d) इनमें से कोई नहीं
उत्तर : (a)
व्याख्या : डीएनए की एक रज्जुक से आनुवंशिक सूचनाओं का आरएनए में प्रतिलिपिकरण करने की प्रक्रिया को ‘अनुलेखन’ कहते हैं।
• यहाँ भी पूरकता का सिद्धांत अनुलेखन प्रक्रम को नियंत्रित करता है जिसमें एडिनीन, थायमीन की जगह पर यूरेसिल के साथ क्षार-युग्म बनाता है।
अतः विकल्प (a) सही उत्तर है।
6. एक अणु जो आनुवंशिक पदार्थ के रूप में कार्य कर सकता है। वह निम्नलिखित में से कौन-सा / से मानदंड को पूरा करता है / करते हैं?
1. यह अपनी प्रतिकृति बनाने में सक्षम है।
2. इसे संरचना व रासायनिक संगठन के आधार पर स्थिर होना चाहिये।
3. इनमें उत्परिवर्तन की संभावना होती है।
उपर्युक्त कथनों में से कौन-सा/से सही है/हैं?
(a) केवल 1 और 2
(b) केवल 2
(c) केवल 2 और 3
(d) 1, 2 और 3
उत्तर : (d)
व्याख्या : एक अणु जो आनुवंशिक पदार्थ के रूप में कार्य कर सकता | है, वह निम्नलिखित मानदंडों को अवश्य पूर्ण करता है-
• यह अपनी प्रतिकृति बनाने में सक्षम है।
• इसे संरचना व रासायनिक संगठन के आधार पर स्थिर होना चाहिये ।
• इनमें उत्परिवर्तन की संभावना होती है जो विकास के लिये आवश्यक है।
• इसे स्वयं ‘मेंडल के लक्षण’ के अनुरूप अभिव्यक्त होना चाहिये ।
• अतः उपर्युक्त तीनों कथन सही है।
अतः विकल्प (d) सही उत्तर है।
7. tRNA के संदर्भ में निम्नलिखित कथनों पर विचार कीजिये-
1. प्रत्येक अमीनो अम्ल के लिये विशिष्ट अंतरण आरएनए tRNA होता है।
2. tRNA को घुलनशील या अनुकूलक RNA भी कहा जाता है।
उपर्युक्त में से कौन-सा/से असत्य है/हैं?
(a) केवल 1
(b) केवल 2
(c) 1 और 2 दोनों
(d) न तो 1 और न ही 2
उत्तर : (d)
व्याख्या : tRNA सबसे छोटा RNA है, जिसमें 75.95 राइबोन्यूक्लियोटाइड पाए जाते है।
• अंतरण आरएनए (tRNA) में एक प्रति प्रकूट (Anticodon) फंदा होता है जिसमें कूट पूरक क्षार मिलते हैं। इसमें एक अमीनो अम्ल स्वीकार्य छोर होता है जिससे यह अमीनो अम्ल से जुड़ जाता है। प्रत्येक अमीनो अम्ल के लिये विशिष्ट t- RNA होते हैं। अतः कथन (1) सही है।
• यह प्रोटीन संश्लेषण के लिये अमीनो अम्ल को कोशिका द्रव्य से राइबोसोम में ले जाते हैं। अतः एक कोशिका में – RNA को घुलनशील (Soluble) या अनुकूलक (Adaptor) RNA भी कहा जाता है। अतः कथन (2) सही है।
अतः विकल्प (d) सही उत्तर है।
8. डीएनए फिंगर प्रिंटिंग तकनीक का प्रारंभिक विकास किसने किया?
(a) वाटसन एवं क्रिक
(b) एलेक जेफ़रेज
(c) फ्लेमिंग
(d) उपर्युक्त में से कोई नहीं
उत्तर : (b)
व्याख्या : डीएनए का प्रयोग कर आनुवंशिक रूप से किसी की पहचान सुनिश्चित करना ही डीएनए फिंगर प्रिंटिंग (DNA Finger Printing) या डीएनए प्रोफाइलिंग कहलाता है। इस प्रक्रिया में किसी व्यक्ति के विभिन्न अवयवों जैसे- बाल, लार, वीर्य, रक्त आदि से उसके डीएनए पदार्थ को एकत्रित किया जाता है तथा उसकी पहचान सुनिश्चित की जाती है।
• डीएनए प्रोफाइलिंग या फिंगरप्रिंटिंग तकनीक का विकास 1984 में ब्रिटिश वैज्ञानिक सर ‘एलेक जेफ़रेज’ ने किया जो लीसेक्टर विश्वविद्यालय के प्राध्यापक थें। वहीं भारत की बात करें तो भारत इस तकनीक का जनक ‘लालजी सिंह’ को माना जाता है।
अतः विकलप (b) सही उत्तर है।
9. निम्नलिखित पर विचार कीजिये –
1. पैतृत्व परीक्षण में सहायक
2. मानव जीनोम के आनुवंशिक नक्शे तैयार करने में लाभदायक में
3. कानूनी उपयोग में एक पहचान अस्त्र के रूप में
4. आपदा या दुर्घटना के दौरान शवों की पहचान करने में
5. आनुवंशिक बीमारियों की पहचान सुनिश्चित करने में
6. कृषि एवं बागवानी के क्षेत्र में बीजों की सही प्रजाति का परीक्षण करने हेतु
उपर्युक्त में से किन-किन क्षेत्रों में डीएनए प्रोफाइलिंग तकनीक का प्रयोग किया जाता हैं ?
(a) केवल 2, 3 और 5
(b) 1, 2, 4 और 6
(c) केवल 3, 4 और 6
(d) उपर्युक्त सभी
उत्तर : (d)
व्याख्या : डीएनए फिंगरप्रिंटिंग या डीएनए प्रोफाइलिंग का इस्तेमाल किसी व्यक्ति की पहचान सुनिश्चित करने के लिये किया जाता है। इस तकनीक का प्रयोग निम्नलिखित क्षेत्रों में किया जाता है-
• किसी बच्चे के सही माता-पिता का पता लगाने में अथवा पैतृक संपत्ति से संबंधित विवादों को सुलझाने में।
• किसी अपराध स्थल से प्राप्त नमूनों के आधार पर अपराधी की पहचान करने में।
• आपदा या दुर्घटना के दौरान शवों की पहचान करने में
• आनुवंशिक बीमारियों की पहचान करने तथा उनसे संबंधित चिकित्सकीय कार्यों को करने के लिये ताकि ऐसी बीमारियों को अगली पीढ़ी में जाने से रोका जा सके।
• कृषि एवं बागवानी के क्षेत्र में बीजों की सही प्रजाति का परीक्षण करने में। यह अधिक उत्तम और वांछित प्रजातियों के विकास में सहायक सिद्ध हो सकता है।
• अतः उपर्युक्त सभी विकल्प सही हैं।
अतः विकल्प (d) सही उत्तर है।

7. क्रम-विकास

1. निम्नलिखित कथनों पर विचार कीजिये-
1. लाइकेन औद्योगिक प्रदूषण के सूचक होते हैं।
2. एक विशेष भू-भौगोलिक क्षेत्र में विभिन्न प्रजातियों के विकास का प्रक्रम एक बिंदु से शुरू होकर अन्य भू-भौगोलिक क्षेत्रों तक प्रसारित होने को अनुकूली विकिरण (Adaptive Radiation) कहा जाता है।
उपर्युक्त कथनों में से कौन-सा/से सत्य है/हैं?
(a) केवल 1
(b) केवल 2
(c) 1 और 2 दोनों
(d) न तो 1 और न ही 2
उत्तर : (c)
व्याख्या : लाइकेन एक प्रकार के मिश्रित जीव (Composite Organism) हैं जो कि एक कवक तथा शैवाल की एक या दो प्रजातियों के साहचर्य के परिणामस्वरूप बनते हैं।
• लाइकेन सल्फर डाइऑक्साइड के प्रति संवेदी (Sensitive) होते हैं। SO2 के कारण इनकी वृद्धि प्रभावित होती है। अतः ये वायु प्रदूषण के अच्छे सूचक (Good Indicators) होते हैं। ये प्रदूषित क्षेत्रों में विलुप्त हो जाते हैं। अतः कथन (1) सही है।
• विकास की वह प्रक्रिया जिसमें किसी भौगोलिक क्षेत्र में नई प्रजातियाँ एक बिंदु से उत्पन्न होती हैं और विभिन्न दिशाओं में विकीर्ण होती हैं, अनुकूली विकिरण (Adaptive Radiation) कहलाता है। यह प्राकृतिक चयन के कारण संभव है। अतः कथन (2) सही है।
अतः विकल्प (c) सही उत्तर है।
2. प्राकृतिक चयन की अवधारणा (Natural Selection) किसने प्रस्तुत किया?
(a) लैमार्क
(b) डार्विन
(c) मेंडल
(d) उपर्युक्त में से कोई नहीं
उत्तर : (b)
व्याख्या: प्राकृतिक चयन की अवधारणा चार्ल्स डार्विन ने प्रस्तुत की, जिसके अनुसार-
• जिस प्रक्रिया द्वारा किसी जनसंख्या में कोई जैविक गुण कम या अधिक हो जाता है उसे प्राकृतिक चयन ( Natural Selection) कहते हैं। यह एक धीमी गति से क्रमशः होने वाली अयादृच्छिक प्रक्रिया है। प्राकृतिक चयन ही क्रमिक विकास की प्रमुख कार्यविधि है।
अतः विकल्प (b) सही उत्तर है।
3. किस वैज्ञानिक ने उत्परिवर्तन के सिद्धांत को प्रस्तुत किया ?
(a) चार्ल्स डार्विन
(b) लैमार्क
(c) ह्यूगो डि-वीस
(d) उपर्युक्त में से कोई नहीं
उत्तर : (c)
व्याख्या: सन् 1901 में ह्यूगो – डि-वीस द्वारा विकास प्रक्रम का एक सिद्धांत (उत्परिवर्तन) प्रतिपादित किया गया।
• इस सिद्धांत के अनुसार आकस्मिक, अपूर्वानुमेय उत्परिवर्तनों के फलस्वरूप ही जीवों की नई प्रजातियों की उत्पत्ति होती है और ये नई प्रजातियाँ प्रारंभ से ही पूर्व प्रजातियों से भिन्न होती हैं।
अतः विकल्प (c) सही उत्तर है।
4. निम्नलिखित पर विचार कीजिये-
1. जननिक पृथक्करण
2. आनुवंशिक पृथक्करण
3. प्राकृतिक चयन
4. भौगोलिक वितरण
उपर्युक्त में से कौन-सी घटना / घटनाएँ औद्योगिक मिलेनिज्म (Industrial Melanism) को दर्शाती है/हैं?
(a) केवल 1
(b) केवल 2 और 3
(c) केवल 1, 2 और 4
(d) केवल 3
उत्तर : (d)
व्याख्या: प्राकृतिक चयन की घटना औद्योगिक मिलेनिज्म को दर्शाती है।
• ऐसी परिघटना जिसमें कुछ प्राणी (जैसे- तितलियाँ, शलभ) औद्योगिक क्षेत्र में अधिक गहरे रंग के हो जाते हैं, जिससे उनका रंग आस-पास के वृक्षों की धुएँ से ढँकी पत्तियों से मेल खा सके, औद्योगिक मिलेनिज्म कहलाती है।
अतः विकल्प (d) सही उत्तर है ।
5. ‘डार्विन फिंच की चोंच’ (एक काली छोटी चिड़िया) एक अच्छा उदाहरण है-
(a) औद्योगिक मिलेनिज्म का
(b) अनुकूली विकिरण का
(c) अभिसारी जैव – विकास का
(d) उपर्युक्त में से कोई नहीं
उत्तर : (b)
व्याख्या : एक स्थान विशेष भू-भौगोलिक भाग विभिन्न प्रजातियों के विकास का एक स्थल से आरंभ होकर अन्य भू-भौगोलिक भागों तक फैल जाने को अनुकूली विकिरण (Adaptive Radiation) कहते हैं।
• इसका प्रमुख उदाहरण पक्षी डार्विन फिंच की चोंच है।
अतः विकल्प (b) सही उत्तर है।
6. कौन – सी एक परिघटना जैव विकास में प्राकृतिक चयन की अवधारणा की पुष्टि करती है-
(a) परजीवी जंतुओं का विकास
(b) क्लोनिंग द्वारा डॉली भेड़ का जन्म
(c) कीटों में कीटनाशक प्रतिरोधकता उत्पन्न होना
(d) उपर्युक्त में से कोई नहीं
उत्तर : (c)
व्याख्या : कीटों में कीटनाशक प्रतिरोधकता का उत्पन्न होना जैव विकास में प्राकृतिक चयन की अवधारणा की पुष्टि करता है।
अतः विकल्प (c) सही उत्तर है।
7. निम्नलिखित पर विचार कीजिये-
1. विलुप्तता
2. विभिन्नता
3. प्रजनन
4. प्रतिस्पर्धा
उपर्युक्त में से कौन – सा / से विकासवाद से युक्त है/हैं?
(a) केवल 2
(b) केवल 1, 3 और 4
(c) केवल 3
(d) केवल 3 और 4
उत्तर : (a)
व्याख्या: समय के साथ-साथ जीवों में क्रमिक परिवर्तन की धारणा विकासवाद कहलाती है।
• चार्ल्स डार्विन के विकासवाद के सिद्धांत में कहा गया है कि विकास प्राकृतिक चयन से होता है । एक प्रजाति के प्राणी शारीरिक विशेषताओं में विभिन्नता दिखाते हैं। ऐसे प्राणी जो अपने पर्यावरण के लिये खराब रूप से अनुकूलित होते हैं, उनके जीवित रहने और प्रजनन करने की संभावना कम होती है।
इसलिये, विकल्प (a) सही उत्तर है।

8. मानव स्वास्थ्य तथा रोग

1. निम्नलिखित कथनों पर विचार कीजिये-
1. चिकनगुनिया एक विषाणुजनित बीमारी है।
2. इटाई – इटाई रोग कैडमियम (cd) के कारण होता है।
3. ब्लैकफुट आर्सेनिक तत्त्वों के कारण होता है।
उपर्युक्त कथनों में से कौन सा/से सही है/हैं?
(a) केवल 2
(b) केवल 1 और 2
(c) केवल 2 और 3
(d) 1, 2 और 3
उत्तर : (d)
व्याख्या : चिकनगुनिया विषाणु से होने वाली मच्छर जनित बीमारी है। यह मच्छर के काटने से मनुष्यों को संक्रमित करता है। यह बुखार और जोड़ों के दर्द का कारण बन सकता है। अतः कथन 1 सही है।
• इटाई – इटाई रोग कैडमियम (cd) के संपर्क में आने के कारण होता है, जो औद्योगीकरण से संबंधित मानवीय गतिविधियों के परिणामस्वरूप उत्पन्न होता है। इटाई – इटाई रोग को अस्थि के गंभीर दर्द के साथ ऑस्टियोमलेशिया की विशेषता द्वारा पहचाना जाता है और यह वृक्क की मलीकीय शिथिलता से जुड़ा हुआ है। इसलिये कथन (2) सही है।
• ब्लैक फुट आर्सेनिक के कारण होने वाली बीमारी है। अतः कथन (3) सही है।
अतः विकल्प (d) सही उत्तर है।
2. निम्नलिखित पर विचार कीजिये-
1. माइक्रोस्पोरम
2. ट्राइकोफाइटॉन
3. एपिडर्मोफाइटॉन
4. पुचेरेरिया
उपर्युक्त में से कौन-सा/से कवक दाद (Ringworm) के लिये उत्तरदायी है/हैं?
(a) केवल 2
(b) केवल 1 और 3
(c) केवल 3 और 4
(d) 1, 2 और 3
उत्तर : (d)
व्याख्या : माइक्रोस्पोरम, ट्राइकोफाइटॉन और एपिडर्मोफाइटॉन आदि वंश के अनेक कवक, दाद (Ringworm) के लिये उत्तरदायी हैं। यह संक्रामक रोग मनुष्यों में बहुत ही सामान्य है। शरीर के विभिन्न भागों जैसे – त्वचा, नाखून और शिरोवल्क (स्कैल्प ) पर सूखी, शल्की विक्षतियाँ (स्केली लीजन) इस रोग के प्रमुख लक्षण हैं।
• इन विक्षतियों में तेज खुजली होती है। ऊष्मा और नमी इन कवकों को त्वचा के वलनों (fold) जैसा कि, ग्रोइन अथवा पादुंगलियों के बीच पनपने में मदद करती है। दाद आमतौर पर मिट्टी से या संक्रमित व्यक्तियों के तौलिये, कपड़े या कंघे तक का प्रयोग करने से हो जाता है।
अतः विकल्प (d) सही उत्तर है।
3. किस विषाणु के कारण चिकनपॉक्स (Chicken Pox) नामक रोग होता है?
(a) वैरीसेला विषाणु
(b) विब्रियोकोलेरा
(c) बेसिलस टाइफोसस
(d) उपर्युक्त में से कोई नहीं
उत्तर : (a)
व्याख्या : चिकनपॉक्स एक संक्रामक रोग है, जो वैरीसैला जोस्टर विषाणु के कारण होता है। इसमें खुजली वाले लाल फफोले होते हैं जो पूरे शरीर में दिखाई देते हैं।
• हैजा एक जीवाणुजनित रोग है, जो विब्रियोकोलेरा नामक जीवाणु द्वारा होता है। जबकि टायफॉइड बुखार बेसिलस टाइफोसस नामक जीवाणु के कारण होता है।
अतः विकल्प (a) सही उत्तर है।
4. निम्नलिखित कथनों पर विचार कीजिये-
1. परपोषी की रोगकारक जीवों से लड़ने की क्षमता जो उसे प्रतिरक्षी तंत्र के कारण मिली है, प्रतिरक्षा (Immunity) कहलाती है।
2. टी-कोशिकाएँ प्रोटीन उत्पन्न करने में बी-कोशिकाओं की सहायता करती हैं।
उपर्युक्त कथनों में से कौन-सा/से सही है/
(a) केवल 1
(b) केवल 2
(c) 1 और 2 दोनों
(d) न तो 1 और न ही 2
उत्तर : (c)
व्याख्या: प्रतिरक्षा के बारे में सबसे पहले रूसी वैज्ञानिक मेनिकिकोव और फ्राँसीसी सूक्ष्म जीवविज्ञानी लुई पाश्चर ने बताया था।
• रोगजनकों की अनुक्रिया में बी- लसीकाणु हमारे रक्त में प्रोटीन की सेना उत्पन्न करते हैं ताकि वे रोगजनकों से लड़ सकें। प्रोटीनें प्रतिरक्षी (Antibotics) कहलाते हैं। टी-कोशिकाएँ खुद तो प्रतिरक्षियों का स्रवण नहीं करती, लेकिन प्रोटीन उत्पन्न करने में बीटा – कोशिकाओं की सहायता करती हैं। इसलिये कथन 1 और 2 सही है।
अतः विकल्प (c) सही उत्तर है।
5. निम्नलिखित कथनों पर विचार कीजिये-
1. पर्यावरण में मौजूद कुछ प्रतिजनों के प्रति प्रतिरक्षा तंत्र की अतिरंजित अनुक्रिया एलर्जी कहलाती है।
2. प्रतिहिस्टैमिन, एड्रिनैलिन और स्टेरॉयडों जैसी औषधियों के प्रयोग से एलर्जी के लक्षण जल्दी घट जाते है।
उपर्युक्त कथनों में से कौन सा/से सत्य है/हैं?
(a) केवल 1
(b) केवल 2
(c) 1 और 2 दोनों
(d) न तो 1 और न ही 2
उत्तर : (c)
व्याख्या : एलर्जी शरीर की उस पदार्थ या उत्पाद के प्रति प्रतिक्रिया है, जिसे वह एक हानिकारक ‘आक्रमणकारी’ के रूप में देखता है। उदाहरण के लिये – पराग, धूल के कण और कोई जानवर यह सभी वह चीजें हैं जिससे | हमें कोई परेशानी नहीं होती। लेकिन कुछ लोगों के इन चीज़ों के संपर्क में आने से व्यक्ति की प्रतिरक्षा प्रणाली प्रतिक्रिया कर सकती है, जिसकी | वजह से कोई शारीरिक समस्या होना शुरू हो जाती है। जैसे- धूल के कणों के संपर्क में आने से छींक आना। अतः वह पदार्थ जो इन प्रतिक्रियाओं का कारण बनते हैं, उन्हें एलर्जी कहा जाता है। अतः कथन (1) सही है।
• प्रतिहिस्टैमिन, एड्रिनैलिन और स्टेरॉयड जैसे औषधियों के प्रयोग से एलर्जी के लक्षणों में कमी आती है। अतः कथन (2) सही है।
अतः विकल्प (c) सही उत्तर है।
6. निम्नलिखित पर विचार कीजिये –
1. अस्थिमज्जा
2. थाइमस
3. प्लीहा
4. बाल्यग्रंथि
उपर्युक्त में से कौन-सा / से लसिकाभ अंग (Lymphoid Organs) है/हैं?
(a) केवल 1
(b) केवल 2, 3 और 4
(c) केवल 1, 2 और 3
(d) केवल 2 और 3
उत्तर : (c)
व्याख्या : लसिकाभ अंग (Lymphoid organs) वे अंग हैं जिसमें लसिकाणुओं की उत्पत्ति, परिपक्वन (Maturation ) और प्रसार (Proliferation) होता है।
• अस्थिमज्जा (Bonemarrow) और थाइमस ऐसे प्राथमिक लसिकाभ अंग है जहाँ अपरिपक्व लसिकाणु, प्रतिजन संवेदनशील लसिकाणुओं में विभेदित होते हैं। परिपक्वन के बाद लसीकाणु प्लीहा (Spleen) लसिका ग्रंथियों, क्षुद्रांत्र के पेयर पैचों और परिशेषिका (Appendix) जैसे द्वितीयक अंगों में चले जाते हैं।
• अस्थिमज्जा मुख्य लसिकाभ अंग है जहाँ लसिकाणुओं समेत सभी रुधिर कोशिकाएँ उत्पन्न होती हैं।
• थाइमस एक सपालि (Lobed) अंग है, जो हृदय के पास उरोस्थि के नीचे स्थित है।
• प्लीहा सेम के आकार का बड़ा अंग है। इसमें मुख्य रूप से लसिकाणु और भक्षकाणु होते हैं। यह लसिका तंत्र का सबसे बड़ा अंग है।
अतः विकल्प (c) सही उत्तर है।
7. एड्स (AIDS) के संदर्भ में निम्नलिखित कथनों पर विचार कीजिये-
1. यह मानव में प्रतिरक्षा न्यूनता विषाणु के कारण होता है।
2. एड्स को टीके द्वारा ठीक किया जा सकता है।
3. भारत में एड्स का टीका उपलब्ध है।
उपर्युक्त कथनों में से कौन सा/से सत्य है/हैं?
(a) केवल 1
(b) केवल 1 और 2
(c) 1, 2 और 3
(d) उपर्युक्त में से कोई नहीं
उत्तर : (a)
8. निम्नलिखित पर विचार कीजिये –
1. कीमोथेरेपी
2. सर्जरी
3. विकिरण चिकित्सा (Radiation Therapy)
4. टीकाकरण
उपर्युक्त में से कौन-सा / से तरीका / तरीके कैंसर के उपचार के लिये सबसे सामान्य है/हैं?
(a) केवल 1
(b) केवल 1, 2 और 4
(c) केवल 3
(d) केवल 1, 2 और 3
उत्तर : (d)
व्याख्या : अधिकांश कैंसर का उपचार सर्जरी, विकिरण चिकित्सा और कीमोथेरेपी के संयोजन से किया जाता है।
• सर्जरी में कैंसर को शरीर के उस हिस्से से जितना हो सके दूर किया जाता है।
• कीमोथेरेपी में दवाओं की मदद से विषाक्त कोशिकाओं को तेजी से विभाजित करने वाली कैंसर की कोशिकाओं को मार दिया जाता है।
• विकिरण चिकित्सा में कैंसर की कोशिकाओं को मारने के लिये शरीर में शक्तिशाली विकिरणों का उपयोग किया जाता है।
अतः विकल्प (d) सही उत्तर है।
9. निम्नलिखित में से कौन-सा / सी तकनीकें आंतरिक अंगों के कैंसर का पता लगाने में उपयोगी है/हैं?
1. रेडियोग्राफी
2. अभिकलित टॉमोग्राफी
3. एम.आर.आई. (MRI)
नीचे दिये गये कूट का प्रयोग कर सही उत्तर चुनिये-
(a) केवल 1 और 2
(b) केवल 3
(c) केवल 2 और 3
(d) 1, 2 और 3
उत्तर : (d)
व्याख्या: आंतरिक अंगों के कैंसर का पता लगाने के लिये विकिरण-चित्रण (रेडियोग्राफी), अभिकलित टॉमोग्राफी और एम.आर.आई. (मैग्नेटिक रेजोनेंस इमेजिंग) जैसी तकनीकें बहुत उपयोगी हैं।
अतः विकल्प (d) सही उत्तर है।
10. निम्नलिखित में से कौन सा विषाणु जनित रोग है ?
(a) हैजा
(b) मलेरिया
(c) पोलियो
(d) उपर्युक्त सभी
उत्तर : (c)
व्याख्या : पोलियो एक संक्रामक रोग है जो पोलियो विषाणु से मुख्यतः छोटे बच्चों में होता है। यह बीमारी बच्चे के किसी भी अंग को जिंदगी भर के लिये कमजोर कर देती है।
• हैजा व्यक्ति द्वारा दूषित भोजन या पानी पीने के कारण होने वाला आँत संबंधी संक्रामक रोग है।
• मलेरिया प्लाज्मोडियम नामक परजीवी के कारण होता है। यह मादा एनाफिलीज मच्छर के काटने से शुरू होता है, जो इस परजीवी को शरीर में छोड़ता है।
अतः विकल्प (c) सही उत्तर है।
11. निम्नलिखित पर विचार कीजिये-
1. मलेरिया
2. डेंगू
3. टायफॉइड
4. चिकनगुनिया
उपर्युक्त में से कौन-सी बीमारी / बीमारियों के लिये ‘विडाल परीक्षण’ (WIDAL Test) किया जाता है?
(a) केवल 2
(b) केवल 1, 2 और 4
(c) केवल 3
(d) केवल 1, 2 और 3
उत्तर : (c)
व्याख्या : साल्मोनेला टाइफी एक रोगजनक जीवाणु है जिससे मानव में टायफॉइड ज्वर होता है। यह रोगजनक जीवाणु आमतौर पर संदूषित भोजन और पानी द्वारा छोटी आँत में घुस जाता है और वहाँ से रक्त द्वारा शरीर के अन्य अंगों में पहुँच जाता है।
• इस रोग के कुछ सामान्य लक्षण हैं- लगातार उच्च ज्वर, कमजोरी, आमाशय में पीड़ा, सिरदर्द और भूख न लगना आदि ।
• टायफॉइड ज्वर की पुष्टि विडाल परीक्षण (WIDAL Test) से हो सकती है।
अतः विकल्प (c) सही उत्तर है।

9. खाद्य उत्पादन में वृद्धि की कार्यनीति

1. निम्नलिखित कथनों पर विचार कीजिये-
1. एक ही नस्ल के पशुओं के मध्य प्रजनन ‘अंतः प्रजनन’ कहलाता है।
2. भिन्न-भिन्न नस्लों के मध्य प्रजनन ‘अंतरा – प्रजनन’ कहलाता है।
उपर्युक्त कथनों में से कौन-सा/से सत्य है/हैं?
(a) केवल 1
(b) केवल 2
(c) 1 और 2 दोनों
(d) न तो 1 और न ही 2
उत्तर : (a)
व्याख्या : एक ही नस्ल के दो पशुओं के मध्य प्रजनन की क्रिया को अन्तः प्रजनन कहते हैं। जैसे- एक ही प्रजाति की गाय और बैल के बीच होने वाला प्रजनन ‘अंतः प्रजनन’ है। अतः कथन 1 सत्य है।
• जबकि भिन्न-भिन्न नस्लों के मध्य प्रजनन कराया जाए तो वह ‘बहि: प्रजनन’ कहलाता है। अतः कथन 2 असत्य है।
अतः विकल्प (a) सही उत्तर है।
2. संकरण (Cross-breeding) के संदर्भ में निम्नलिखित कथनों पर विचार कीजिये –
1. इसमें एक नस्ल की अथवा भिन्न-भिन्न नस्लों अथवा भिन्न प्रजातियों की व्यष्टियाँ भाग लेती हैं।
2. यह समयुग्मता (Homozygosity) को बढ़ावा देता है।
3. इस विधि द्वारा पशुओं की अनेक नई नस्लों का विकास हुआ है।
उपर्युक्त कथनों में से कौन-सा/से सही है/हैं?
(a) कंवल 2
(b) केवल 1 और 2
(c) केवल 2, 3 और 4
(d) केवल 3
उत्तर : (d)
व्याख्या: संकरण (Cross-breeding) में एक नस्ल के श्रेष्ठ नर का दूसरी नस्ल की श्रेष्ठ मादा के साथ संगम कराया जाता है।
• संकरण दो विभिन्न नस्लों के वांछनीय गुणों के संयोजन में सहायक होता है। इसी विधि द्वारा पशुओं की अनेक नई नस्लों का विकास हुआ है। अतः केवल कथन 3 सही है।
• अंत: प्रजनन (Inbreeding) समयुग्मता को बढ़ाता है।
• बहिः प्रजनन (Out breeding) में एक नस्ल की अथवा भिन्न-भिन्न नस्लों अथवा भिन्न प्रजातियों की व्यष्टियाँ भाग लेती हैं।
अत: विकल्प (d) सही है।
3. कतला, रोहू तथा कॉमन कार्प जैसी मछलियाँ पाई जाती हैं-
(a) लवणीय जल में
(b) अलवणीय जल में
(c) लवणीय और अलवणीय दोनों जल में
(d) उपर्युक्त में से कोई नहीं
उत्तर : (b)
व्याख्या : कतला, रोहू तथा कॉमन कार्प जैसी मछलियाँ सामान्यतः अलवणीय जल में पाई जाती हैं।
• कतला एक प्रसिद्ध और जल्दी बढ़ने वाली कार्प है। यह प्रकृति में गैर-हिंसक होती है। यह पानी की ऊपरी सतह से भोजन लेती है और इसके विकास के लिये किनारे और सतह की वनस्पति बहुत उपयोगी होती है। स्वादिष्ट भोजन और उच्च प्रोटीन सामग्री के कारण इसकी मांग अपेक्षाकृत अधिक होती है।
• रोहू मछली मीठे पानी के स्रोतों में पाई जाती है। रोहू के शरीर पर अन्य मछलियों की तरह शल्क पाए जाते हैं। यह एक पृष्ठवंशी (Vertebrates) हड्डी युक्त कंकाल वाली मछली है।
• कॉमन कार्प जल्दी बढ़ने वाली सर्वाहारी मछली है जो जल में पाई जाती है। यह खुद को बदलते मौसम की स्थितियों के साथ आसानी से ढाल लेती है, इसलिये इसे विश्व के दोनों गर्म और ठंडे क्षेत्रों में पाला जा सकता है।
अत: विकल्प (b) सही उत्तर है।
4. निम्नलिखित पर विचार कीजिये –
1. गेहूँ का भूरा किट्ट (Brown Rust of Wheat)
2. गन्ने का रेड रॉट रोग (Red rot of Sugar Cane)
3. तंबाकू मोजेक (Tobacco Mosaic)
4. क्रूसीफर का ब्लैक रॉट (Black Rot of Crucifers)
उपर्युक्त में से कौन-से पादपों में कवकों द्वारा उत्पन्न रोगों के उदाहरण हैं?
(a) केवल 1 और 3
(b) केवल 2 और 3
(c) केवल 1 और 2
(d) केवल 1, 2 और 4
उत्तर : (c)
व्याख्या: पादपों में कवकों द्वारा उत्पन्न होने वाले रोगों के उदाहरण निम्नलिखित हैं-
• गेहूँ का भूरा किट्ट (Brown Rust of Wheat)
• गन्ने का रेड रॉट (लाल सड़न) रोग (Red Rot of Sugarcane)
• आलू पछेती अंगमारी ( Late blight of Potato)
• तबांकू मोजेक (Tobacco Mosaic) विषाणु द्वारा उत्पन्न रोग है।
• क्रूसीफर का ब्लैक रॉट (Black Rot of Crucifers) जीवाणु द्वारा उत्पन्न रोग है।
अतः विकल्प (c) सही उत्तर है।
5. एकल कोशिका प्रोटीन के संदर्भ में निम्नलिखित कथनों पर विचार कीजिये-
1. यह संपूर्ण पोषक आहार है जिसमें केवल प्रोटीन और विटामिन पाए जाते है।
2. इसका प्रयोग पर्यावरणीय प्रदूषण को भी कम करता है ।
उपर्युक्त कथनों में से कौन-सा/से सही है/हैं?
(a) केवल 1
(b) केवल 2
(c) 1 और 2 दोनों
(d) न तो 1 और न ही 2
उत्तर : (b)
व्याख्या: पशु और मानव के पोषण के लिये सूक्ष्म जीवों का प्रोटीन स्रोत के रूप में उपयोग करना एकल कोशिका प्रोटीन कहलाता है। जैसे -मशरूम स्पिरुलीना
• यह संपूर्ण पोषक आहार है जिसमें प्रोटीन, कार्बोहाइड्रेड, वसा, विटामिन तथा खनिज लवण पाए जाते है। अतः कथन 1 सही नहीं है।
• इससे प्रदूषण कम होता है। अतः केवल कथन 2 सही है।
अतः विकल्प (b) सही उत्तर है।
6. नॉर्मन बोरलॉग किससे संबंधित हैं?
(a) सफेद क्रांति
(b) हरित क्रांति
(c) नीली क्रांति
(d) पीली क्रांति
उत्तर : (b)
व्याख्या: नॉर्मन बोरलॉग एक अमेरिकी कृषि विज्ञानी थे। इन्हें हरित क्रांति का जनक माना जाता है।
• इन्होंने मैक्सिको स्थित इंटरनेशनल सेंटर फॉर व्हीट एंड मेंज में गेहूँ की अर्द्ध-वामन किस्म का विकास किया। उनके नवीन प्रयोगों ने अनाज की समस्या से जूझ रहे भारत सहित अनेक विकासशील देशों में हरित क्रांति का प्रवर्तन करने में महत्त्वपूर्ण योगदान दिया ।
अतः विकल्प (b) सही उत्तर है।
7. पौधों में ऊतक संवर्द्धन (Tissue Culture) के संदर्भ में निम्नलिखित कथनों पर विचार कीजिये-
1. इसका प्रयोग पौधों के व्यावसायिक उत्पादन के लिये किया जाता है।
2. पादपों में इस तकनीक द्वारा रोग प्रतिरोधी, कीट रोधी तथा सूखा प्रतिरोधी किस्मों को उत्पादित किया जा सकता है।
उपर्युक्त में से कौन-सा/से सही है/हैं?
(a) केवल 1
(b) केवल 2
(c) 1 और 2 दोनों
(d) न तो 1 और न ही 2
उत्तर : (c)
व्याख्या : ऊतक संवर्द्धन उपयुक्त विकास माध्यम में पौधे के ऊतक के एक छोटे से टुकड़े से या पौधे की बढ़ती युक्तियों से कोशिकाओं को हटाकर नए पौधों के उत्पादन की एक प्रक्रिया है।
• इसका प्रयोग व्यावसायिक उत्पादन एवं अनुसंधान के लिये किया जाता है। अतः कथन 1 सही है।
• ऊतक संवर्द्धन द्वारा उत्पादित नए पौधे रोग मुक्त होते हैं, इस तकनीक द्वारा रोग प्रतिरोधी, कीट रोधी तथा सूखा प्रतिरोधी किस्मों को उत्पादित किया जा सकता है। अतः कथन 2 भी सही है।
अतः विकल्प (c) सही उत्तर है।

10. मानव कल्याण में सूक्ष्मजीव

1. निम्नलिखित पर विचार कीजिये –
1. जीवाणु
2. बेकर्स यीस्ट
3. स्यूडोमोनास
4. राइजोपस स्टोलोनीफर
उपर्युक्त में से ब्रेड बनाने में किसका इस्तेमाल किया जाता है?
(a) केवल 1 और 4
(b) केवल 2
(c) केवल 2, 3 और 4
(d) उपर्युक्त में कोई नहीं
उत्तर : (b)
व्याख्या : इनवर्टेज एन्जाइम सैकरोमाइसीज सैरीवीसी से प्राप्त किया जाता है और ब्रेड निर्माण में उपयोग किया जाता है इसलिये यह ‘बेकर्स यीस्ट’ कहलाता है।
अत: विकल्प (b) सही उत्तर है।
2. निम्नलिखित में से किस जीवाणु/किन जीवाणुओं का उपयोग दूध से दही बनाने में होता है?
1. स्ट्रैप्टोकोकस
2. लैक्टोबैसिलस
3. नाइट्रोसोमोनास
नीचे दिये गए कूट में से सही उत्तर चुनिये-
(a) केवल 1
(b) केवल 1 और 2
(c) केवल 2
(d) 1, 2 और 3
उत्तर : (c)
व्याख्या: दही में लैक्टोबैसिलस नाम का बैक्टीरिया पाया जाता है। दूध को जब हम 30°-40°C पर गर्म करके इसमें थोड़ा सा पुराना दही मिला देते हैं तो उसमें मौजूद लैक्टोबैसिलस नामक जीवाणु बढ़ने लगते है।
• ये जीवाणु दूध में मौजूद लैक्टोस (Lactose) को लैक्टिक अम्ल (Lactic Acid) में बदल देते हैं।
अतः विकल्प (c) सही उत्तर है।
3. निम्नलिखित कथनों पर विचार कीजिये-
1. सूक्ष्मजीवियों का प्रयोग किण्वित मत्स्य, सोयाबीन तथा बाँस प्ररोह आदि के भोजन तैयार करने में किया जाता है।
2. ‘टोडी’ उत्तर भारत का एक पारंपरिक पेय पदार्थ है।
उपर्युक्त कथनों में से कौन सा/से असत्य है/हैं?
(a) केवल 1
(b) केवल 2
(c) 1 और 2 दोनों
(d) न तो 1 और न ही 2
उत्तर : (b)
व्याख्या: पारंपरिक पेय तथा आहारों की एक बड़ी संख्या सूक्ष्मजीवियों द्वारा किण्वित कराकर तैयार की जाती है। दक्षिण भारत के कुछ भागों में एक पारंपरिक पेय ‘टोडी’ है। इसे ताड़वृक्ष के तने के स्त्राव को किण्वित कराकर तैयार किया जाता है। अतः कथन (2) असत्य है।
• जबकि सूक्ष्मजीवियों का प्रयोग किण्वित मत्स्य (मछली), सोयाबीन तथा बाँस प्ररोह आदि के भोजन तैयार करने में किया जाता है। अतः कथन (1) सत्य है।
अतः विकल्प (b) सही उत्तर है ।
4. निम्नलिखित पर विचार कीजिये-
1. वाइन
2. बियर
3. व्हिस्की
4. ब्रांडी
उपर्युक्त में से किन पदार्थों का निर्माण बिना आसवन (Without distillation) की क्रिया के होता है/हैं?
(a) केवल 3
(b) केवल 1 और 2
(c) केवल 1, 2 और 4
(d) उपर्युक्त सभी
उत्तर : (b)
व्याख्या: जब दो द्रवों के क्वथनांक में अंतर अधिक होता है, तो उसके मिश्रण को आसवन विधि से पृथक करते हैं। अर्थात् यह द्रवों के मिश्रण को अलग करने की विधि है। इसका प्रथम भाग वाष्पीकरण (Vapourisation) और दूसरा भाग संघनन (Condensation) कहलाता है।
• वाइन और बियर का उत्पादन बिना आसवन के जबकि व्हिस्की, ब्रांडी और रम किण्वित रस के आसवन द्वारा तैयार किये जाते हैं।
अतः विकल्प (b) सही उत्तर है।
5. निम्नलिखित में से कौन गोबर गैस का प्रमुख घटक है/हैं?
1. ब्यूटेन
2. अमोनिया
3. मीथेन
नीचे दिये गए कूट का प्रयोग कर सही उत्तर चुनिये-
(a) केवल 1
(b) केवल 1 और 2
(c) केवल 3
(d) 1, 2 और 3
उत्तर : (c)
व्याख्या: गोबर गैस को बायोगैस के रूप में भी जाना जाता है। इसका उत्पादन जीवों और पौधों से जैविक कचरे के अवायवीय अपघटन के माध्यम से किया जाता है।
• यह मुख्य रूप से मीथेन और कार्बन डाइऑक्साइड का मिश्रण होता है। मीथेन बायोगैस का एक प्रमुख घटक है।
अतः विकल्प (c) सही उत्तर है।
6. निम्नलिखित में से कौन जैव गैस उत्पादन में प्रयुक्त जीवाणु का समूह है?
1. आर्गेनोट्रॉफ
2. मेथेनोट्रॉफ
3. मीथेनोजेन
नीचे दिये गए कूट में से सही उत्तर चुनिये-
(a) केवल 1
(b) केवल 1 और 2
(c) केवल 3
(d) 1, 2 और 3
उत्तर : (c)
7. निम्नलिखित में से कौन-सा/से जैव उवर्रक (Biofertilizer) है/हैं?
1. माइकोराइजा
2. एजोटोबैक्टर
3. स्यूडोमोनास
नीचे दिये गए कूट का प्रयोग कर सही उत्तर चुनिये-
(a) केवल 1
(b) केवल 1 और 2
(c) केवल 3
(d) 1, 2 और 3
उत्तर : (d)
व्याख्या : जैव उर्वरक जीवित सूक्ष्मजीवों से समृद्ध होते हैं। बीज, मिट्टी या जीवित पौधों पर इनका छिड़काव किये जाने से मिट्टी के पोषक तत्त्वों में वृद्धि होती है। इसमें विभिन्न प्रकार के कवक, मूल जीवाणु या अन्य सूक्ष्मजीव होते हैं जो मेजबान पौधों के साथ पारस्परिक रूप से लाभप्रद या सहजीवी संबंध बनाते है क्योंकि इनकी उत्पत्ति मिट्टी में होती है।
• राइजोबियम (Rhizobium), एजोटोबैक्टर (Azotobacter), स्यूडोमोनॉस (Pseudomonas), माइकोराइजा आदि जैव उर्वरक हैं।
अत: विकल्प (d) सही है।
8. निम्नलिखित में से कौन-सा / से मिट्टी को उपजाऊ बनाने में सहायक है/हैं?
1. कीटनाशक
2. जैविक खाद
3. यीस्ट
नीचे दिये गए कूट का प्रयोग कर सही उत्तर चुनिये-
(a) केवल 1
(b) केवल 1 और 2
(c) केवल 2
(d) 1, 2 और 3
उत्तर : (c)
व्याख्या : जीव-जंतुओं के मल मूत्र, शरीर अवशेष, फार्म पर उगाई फसलों एवं उद्योगों के उत्पादों आदि के विघटन से निर्मित पदार्थ को जैविक खाद (Organic Manure) कहते हैं।
• इससे भूमि की अवस्था में सुधार होता है और मृदा में वायु संचार बढ़ता है।
• इसके प्रयोग से विभिन्न रसायनों के दुष्प्रभाव से भूमि, पर्यावरण व कृषि उत्पाद को बचाया जा सकता है।
अतः विकल्प (c) सही उत्तर है।
9. यदि एक नदी सीवेज से दूषित हो जाए तो मछलियाँ मर जाएंगी क्योंकि-
(a) जल में O2 की मात्रा कम हो जाती है।
(b) जल के दुर्गंध के कारण
(c) सीवेज में उपस्थित रोगाणु के कारण
(d) ठोस पदार्थों के श्लेष्म में फँसे होने के कारण
उत्तर : (a)
व्याख्या:
• यदि नदियों, तालाबों एवं झीलों का पानी दूषित हो जाता है, तो मछलियाँ मरने लगती हैं। पानी में घुली ऑक्सीजन की मात्रा में कमी होने के कारण मछलियाँ लगातार मरने लगती हैं।
अतः विकल्प (a) सही उत्तर है।

11. जैव प्रौद्योगिकी- सिद्धांत व प्रक्रम

1. निम्नलिखित कथनों पर विचार कीजिये-
1. पादप एवं जीवों के लिये लैंगिक जनन, अलैंगिक जनन से अधिक लाभकारी है।
2. पौधों एवं जंतुओं के जनन हेतु उपयोग में लाई जाने वाली संकरण विधि से केवल वांछित जीन का ही समावेश होता है|
उपर्युक्त में से कौन – सा /से कथन सत्य है/हैं?
(a) केवल 1
(b) केवल 2
(c) 1 और 2 दोनों
(d) न तो 1 और न ही 2
उत्तर : (a)
व्याख्या: केवल पहला कथन सत्य है। अलैंगिक जनन आनुवंशिक सूचनाओं को परिरक्षित रखता है, जबकि लैंगिक जनन द्वारा विभिन्नता व विशिष्ट आनुवंशिक व्यवस्था के संयोजन के प्रतिपादन का अवसर मिलता है जो जीव या आबादी हेतु लाभकारी हो सकता है। लैंगिक जनन से विभिन्नता उत्पन्न होती है। अतः अलैंगिक जनन की अपेक्षा लैंगिक जनन अधिक फायदेमंद होता है।
• दूसरा कथन असत्य है। परंपरागत संकरण की विधियाँ जो पौधों एवं जंतुओं के जनन में उपयोगी हैं, इनके द्वारा वांछित जीन के साथ-साथ अवांछित जीन का समावेश व गुणन हो जाता है। उपर्युक्त कमियों को दूर करने हेतु आनुवंशिक इंजीनियरिंग तकनीकों में जीन क्लोनिंग एवं जीन स्थानांतरण का उपयोग कर पुनर्योगज डीएनए (रीकॉम्बीनेंट डीएनए) का निर्माण किया जाता है।
2. पुनर्योगज डीएनए तकनीक ( रिकॉम्बीनेंट डीएनए टेक्नोलॉजी) के अंतर्गत डीएनए को विभिन्न जगहों से काटने के लिये उपयोग किया जाता है-
(a) बीटा कैरोटीन
(b) प्रतिबंधन एंजाइम (रिस्ट्रिक्सन एंजाइम )
(c) प्लाज्मिड
(d) कैटैलेज
उत्तर : (b)
व्याख्या: पुनर्योगज डीएनए तकनीक के अंतर्गत डीएनए को विशिष्ट जगहों से काटने के लिये आण्विक कैंची कहे जाने वाले ‘प्रतिबंधन एंजाइम’ (रिस्ट्रिक्सन एंजाइम ) का प्रयोग किया जाता है।
3. जेनेटिक इंजीनियरिंग में निम्न में से किसका प्रयोग होता है ?
(a) प्लास्टिड
(b) प्लाज्मिड
(c) माइटोकॉण्ड्रिया
(d) राइबोसोम
उत्तर : (b)
व्याख्या : जेनेटिक इंजीनियरिंग में प्लाज्मिड का प्रयोग होता है। प्राकृतिक रूप से प्लाज्मिड, बैक्टेरिया कोशिका तथा कुछ यूकैरियाट्स में पाए जाते हैं। प्लाज्मिड द्विकुंडलीय छोटा गोलाकार डीएनए अणु होता है जो कोशिका के क्रोमोसोमल डीएनए से अलग होता है। प्लाज्मिड डीएनए संवाहक ( वेक्टर) की तरह कार्य करता है। जिस तरह से मच्छर, कीट संवाहक के रूप में मलेरिया परजीवी को मनुष्य के शरीर में स्थानांतरित करता है, ठीक उसी तरह प्लाज्मिड को संवाहक के रूप में प्रयोग कर विजातीय डीएनए के खंड को परपोषी जीवों में पहुँचाया जाता है।
• जेनेटिक इंजीनियरिंग में आनुवंशिक पदार्थों (डीएनए या आरएनए) के रसायन में परिवर्तन कर इसे परपोषी जीवों (होस्ट आर्गेनिज़्म) प्रवेश कराकर इसके समलक्षणी (फीनोटाइप) में परिवर्तन करते हैं।
• जेनेटिक इंजीनियरिंग (आनुवंशिक इंजीनियरिंग) तकनीकों में जीन क्लोनिंग एवं जीन स्थानांतरण का उपयोग कर पुनर्योगज डीएनए (रिकॉम्बीनेंट डीएनए) का निर्माण किया जाता है, जिससे बिना अवांछित जीनों के केवल एक या एक से अधिक वांछित जीन को चुने हुए जीवों में स्थानांतरित करते हैं।
4. पुनर्योगज डीएनए तकनीक ( रिकॉम्बीनेंट डीएनए टेक्नोलॉजी) जीनों को स्थानांतरित होने देता है-
1. पौधों की विभिन्न जातियों में ।
2. जंतुओं से पौधों में।
3. सूक्ष्मजीवों से उच्चतर जीवों में ।
नीचे दिये गए कूटों का प्रयोग कर सही उत्तर चुनिये-
(a) केवल 1
(b) केवल 2 और 3
(c) केवल 1 और 3
(d) 1, 2 और 3
उत्तर : (d)
व्याख्या: रिकॉम्बीनेंट डीएनए टेक्नोलॉजी की सहायता से जीनों को पौधों की विभिन्न प्रजातियों, जंतुओं से पौधों या पौधों से जंतुओं में तथा सूक्ष्मजीवों से उच्चतर जीवों में स्थानांतरित किया जा सकता है।
• जीनों को पादप और जंतुओं में स्थानांतरित करना मानव ने जीवाणुओं और विषाणुओं से सीखा है, जिन्हें यह बात चिरकाल से पता थी। जीवाणु और विषाणु जानते थे कि सुकेंद्रीय (यूकैरियोटिक) कोशिकाओं को रूपांतरित करने के लिये जीनों का कैसे उपयोग किया जाए और वे जैसा चाहते हैं वैसे करने के लिये जीनों को बाध्य करते हैं।
5. निम्नलिखित में से कौन-सा पृथ्वी पर पाए जाने वाले सभी जीव का आनुवंशिक पदार्थ है?
(a) एमीलाइड प्रीकर्सर प्रोटीन
(b) अमीनो अम्ल
(c) न्यूक्लिक अम्ल
(d) एड्रिनलीन
उत्तर : (c)
व्याख्या: पृथ्वी पर पाए जाने वाले सभी जीवों का आनुवंशिक पदार्थ न्यूक्लिक अम्ल है। अधिकांश जीवों में यह डिऑक्सीराइबो न्यूक्लिक अम्ल (DNA) है।
6. आनुवंशिक इंजीनियरिंग के अंतर्गत बायोलिस्टीक या “जीन गन” का प्रयोग किया जाता है- 
(a) डीएनए को विभिन्न जगहों से काटने के लिये ।
(b) जीन या डीएनए खंड को काटने के बाद उनको आकार अनुसार अलग करने के लिये ।
(c) मानव के लिये उपयोगी सूक्ष्मजीवों का क्लोन बनाने के लिये ।
(d) पौधों की कोशिकाओं में पुनर्योगज डीएनए का बलपूर्वक प्रवेश कराने के लिये।
उत्तर : (d)
व्याख्या: परपोषी कोशिकाओं में विजातीय डीएनए को प्रवेश कराने के लिये कई विधियों का प्रयोग किया जाता है। सूक्ष्म अंत:क्षेपण (माइक्रोइंजेक्शन) विधि में पुनर्योगज डीएनए को सीधे जंतु कोशिका केंद्रक के भीतर अंत:क्षेपित किया जाता है। दूसरी विधि है जो पौधों के लिये उपयोगी है। पौधों की कोशिकाओं पर पुनर्योगज डीएनए से विलेपित, स्वर्ण या टंगस्टन के उच्च वेग सूक्ष्म कणों से बमबारी करते हैं जिसे ‘बायोलिस्टीक’ या ‘जीन गन’ कहते हैं।
7. निम्नलिखित कथनों पर विचार कीजिये-
1. डीएनए को प्रतिबंधन एंजाइम (रिस्ट्रिक्सन एंजाइम) द्वारा काटने के लिये दूसरे वृहद् – अणुओं से मुक्त शुद्ध रूप में होना चाहिये ।
2. जीन, हिस्टोन प्रोटीन के साथ गुथे हुए डीएनए पर स्थित होते हैं।
उपर्युक्त में से कौन-सा/से कथन सत्य है/हैं?
(a) केवल 1
(b) केवल 2
(c) 1 और 2 दोनों
(d) न तो 1 और न ही 2
उत्तर : (c)
व्याख्या : उपर्युक्त दोनों कथन सत्य हैं।
• डीएनए को प्रतिबंधन एंजाइम (रिस्ट्रिक्सन एंजाइम) द्वारा काटने के लिये दूसरे वृहद् – अणुओं से मुक्त शुद्ध रूप में होना चाहिये। डीएनए झिल्लियों से घिरा रहता है, इसलिये कोशिका को तोड़कर खोलना पड़ेगा ताकि डीएनए दूसरे वृहद् अणुओं जैसे- आरएनए, प्रोटीन, बहुशर्करा, लिपिड के साथ मोचित (रिलीज) हो सके। यह तभी संभव है जब जीवाणु कोशिका / पादप या जंतु ऊतक, लाइसोजाइम (जीवाणु), सेलुलोज (पादप कोशिका), काइटिनेज (कवक) जैसे एंजाइम द्वारा संसाधित किये जाते हैं।
• जीन डीएनए के लंबे अणुओं पर स्थित होते हैं व हिस्टोन जैसे प्रोटीनों के साथ गुँथे रहते हैं।
8. जेनेटिक इंजीनियरिंग के संदर्भ में निम्नलिखित कथनों पर विचार कीजिये-
1. जेनेटिक इंजीनियरिंग में एक विशिष्ट जीन को प्राप्तकर्त्ता जीव में प्रत्यारोपित किया जा सकता है।
2. यह तकनीकी मलेरिया और डेंगू बुखार जैसी कुछ बीमारियों को रोकने में मदद करेगी।
उपर्युक्त कथनों में से कौन सा/से सही है/हैं?
(a) केवल 1
(b) केवल 2
(c) 1 और 2 दोनों
(d) न तो 1 और न ही 2
उत्तर : (c)
व्याख्या : जेनेटिंग इंजीनियरिंग के माध्यम से वैज्ञानिक वांछनीय जीन को एक पौधे या जीव से दूसरे पौधे या जीव में स्थानांतरित करने में सक्षम होते हैं।
• यह एक ऐसी तकनीक है, जिसमें एक विशिष्ट जीन को चुना जा सकता है और प्राप्तकर्त्ता जीव में प्रत्यारोपित किया जा सकता है। अत: कथन (1) सही है।
• जेनेटिक इंजीनियरिंग के माध्यम से एक कार्यात्मक जीन के साथ दोषपूर्ण जीन को प्रतिस्थापित करके आनुवंशिक विकार भी दूर किये जा सकते है। यह तकनीकी मलेरिया और डेंगू बुखार जैसी कुछ बीमारियों के प्रसार को रोकने में मदद करेगी। अतः कथन (2) भी सही है।
अतः विकल्प (c) सही उत्तर है।
9. निम्नलिखित कथनों पर विचार कीजिये-
1. आनुवंशिक रूप से समान कोशिकाओं अथवा जीवों के समूह को क्लोन (Clone) कहते हैं।
2. एक गुणसूत्र में एक विशिष्ट डीएनए अनुक्रम होता है, जो प्रतिकृतीयन (Replication) के लिये उत्तरदायी होता है।
उपर्युक्त कथनों में से कौन-सा/से सही है/हैं?
(a) केवल 1
(b) केवल 2
(c) 1 और 2 दोनों
(d) न तो 1 और न ही 2
उत्तर : (c)
व्याख्या: क्लोनिंग का तात्पर्य है अलैंगिक विधि से एक जीव से दूसरा जीव तैयार करना । इस विधि से उत्पादित क्लोन अपने जनक से शारीरिक और आनुवंशिक रूप में समरूप होते हैं। अर्थात् किसी जीव का प्रतिरूप | तैयार करना ही क्लोनिंग है। अतः कथन (1) सही है।
• एक गुणसूत्र में एक विशिष्ट डीएनए अनुक्रम होता है जिसे प्रतिकृतीयन (Replication) उत्पत्ति कहते हैं, जो प्रतिकृतीयन के लिये उत्तरदायी होता है। किसी भी जीव में किसी विजातीय खंड के गुणन हेतु इसे गुणसूत्र का अंग होना आवश्यक है जिसमें एक विशिष्ट अनुक्रम मिलता है, जिसे ‘प्रतिकृति का मूल (Origin of Replication)’ कहते हैं। अतः कथन (2) सही है।
अतः विकल्प (c) सही उत्तर है।
10. निम्नलिखित पर विचार कीजिये-
1. समुद्री घास
2. मक्का
3. हाइड्रिला
4. फर्न
उपर्युक्त में से किससे ऐगारोज जेल प्राप्त किया जाता हैं?
(a) केवल 1
(b) केवल 1, 2 और 3
(c) केवल 2 और 4
(d) उपर्युक्त सभी
उत्तर : (a)
व्याख्या: आजकल सामान्य रूप से उपयोग किया जाने वाला ऐगारोज एक प्राकृतिक बहुलक (पॉलीमर) है, जिसे समुद्री घास (Sea Weeds) से प्राप्त किया जाता है।
अतः विकल्प (a) सही उत्तर है।
11. प्रतिबंधित एंडोन्यूक्लिएज के उपयोग के संदर्भ में निम्नलिखित कथनों पर विचार कीजिये-
1. यह औजार ( कैंची) की तरह प्रयोग होता है।
2. यह जिस स्थल पर जीन जाकर जुड़ेगा उस स्थल की पहचान करने में सहायक होता है।
3. यह डीएनए अणु को सूक्ष्म खंडों में काटने पर विशिष्ट जीन की स्थिति का ज्ञान करने में सहायक होता है।
उपर्युक्त में से कौन-सा/से प्रतिबंधित एन्डोन्यूक्लिएज (Restricted Endonuclease) के अनुप्रयोग है / हैं?
(a) केवल 1
(b) केवल 2 और 3
(c) केवल 1 और 2
(d) 1, 2 और 3
उत्तर : (d)
व्याख्या : प्रतिबंधित एन्डोन्यूक्लिएज एक औजार ( कैंची) की तरह प्रयोग होता है जो डीएनए खंड में किन्हीं विशिष्ट अनुक्रमों पर कट (nick) लगा कर उन्हें विखंडित कर देता है।
प्रतिबंधित एन्डोन्यूक्लिएज के अनुप्रयोग-
• यह एक औज़ार (आण्विक कैंची) (Moleular Scissors) की तरह होता है, जो डीएनए खंड में किसी विशिष्ट अनुक्रमों पर कट लगा कर उन्हें विखंडित कर देता है।
• बाह्य डीएनए खंड व वाहक के डीएनए को एक ही प्रतिबंधित एंजाइम से काटने पर वह एक-दसरे के पूरक होने से आसानी से जुड़ (ligate) जाते हैं।
• जिस स्थल पर जीन आकर जुड़ेगा उस स्थल की पहचान करने में सहायक होते है।
• डीएनए अणु को सूक्ष्म खंडों में काटने पर विशिष्ट जीन की स्थिति को जानने में सहायक है।
अतः विकल्प (d) सही उत्तर है।
12. निम्नलिखित पर विचार कीजिये-
1. लाइगेज
2. एक्सोन्यूक्लिएज
3. एंडोन्यूक्लिएज
4. ई. कोलाई
उपर्युक्त में से कौन-सा / से प्रतिबंधित एंजाइम है / हैं ?
(a) केवल 2
(b) केवल 2 और 3
(c) केवल 2, 3 और 4
(d) केवल 1, 3 और 4
उत्तर : (b)
13. निम्नलिखित में से पीसीआर (पॉलिमरेज श्रृंखला अभिक्रिया) का उपयोग किया जा सकता है-
1. डीएनए के किसी खास संग्रह के प्रबंधन के लिये
2. एन्जाइम में प्रवर्द्धन के लिये
3. प्रोटीन के प्रवर्द्धन के लिये
नीचे दिये गए कूट का प्रयोग कर सही उत्तर चुनिये-
(a) केवल 1
(b) केवल 2 और 3
(c) केवल 3
(d) 1, 2 और 3
उत्तर : (a)
व्याख्या: पॉलिमरेज श्रृंखला अभिक्रिया (Polymerase Chain Reaction) के द्वारा जीवों से पृथक किये गए डीएनए खंडों की कई प्रतिलिपियाँ (Copies) प्राप्त की जाती हैं।
इसके अनुसार-
• पीसीआर के द्वारा रक्त सीरम में किसी रोगजनक की सूक्ष्म मात्रा का पता लगाया जा सकता है।
• जीन मैनिपुलेशन (Gene Manipulation) और डीएनए लाइब्रेरी निर्माण के लिये डीएनए खंडों का प्रचार व प्रबंधन करने में भ्रूण *लिंग निर्धारण तथा लिंग संबंधी विकारों का पता लगाने में सहायक हैं।
• आनुवंशिक रोगों को ज्ञात करने में इसका उपयोग किया जाता है।
• डीएनए के अनुक्रम (DNA Sequencing) को ज्ञात करने में भी पीसीआर का उपयोग किया जाता है।
• जीवाणु एवं विषाणु से होने वाले संक्रमण तथा कैंसर आदि का निवारण भी पीसीआर की सहायता से किया जाता सकता है।
अतः विकल्प (a) सही उत्तर है।
14. यदि कोई प्रोटीन कूटलेखन (Encoding) जीन किसी विषमजात परपोषी (Heterologous Host) में अभिव्यक्त होता है, तो इसे कहते हैं-
(a) डीएनए पृथक्करण
(b) संवाहक डीएनए
(c) पुनर्योगज प्रोटीन (Recombinant Protein)
(d) उपर्युक्त में से कोई नहीं
उत्तर : (c)
व्याख्या:
• यदि कोई प्रोटीन कूटलेखन जीन किसी विषमजात परपोषी में अभिव्यक्त होता है, तो इसे पुनर्योगज प्रोटीन (Recombinant Protein) कहते हैं।
• इसका उपयोग आमतौर पर दवा उत्पादों, दवा वितरण के लिये प्रोटीन आधारित रोग उपचार के लिये प्रतिजैविकों के निर्माण आदि के लिये किया जाता है।
अतः विकल्प (c) सही उत्तर है।

12. जैव प्रौद्योगिकी एवं उसके उपयोग

1. जेनेटिकली मोडीफाइड (आनुवंशिक रूपांतरित) फसलों के संबंध में कौन-सा/से कथन सत्य है/हैं?
1. ये फसलें सूखे, ठंड, लवणता एवं ताप के प्रति अधिक सहिष्णु होती हैं।
2. इन फसलों की खनिज उपयोग क्षमता अधिक होती है।
3. इन फसलों की कीटों के प्रति प्रतिरोधी क्षमता अधिक होती है।
कूट :
(a) केवल 1
(b) केवल 2 और 3
(c) केवल 1 और 3
(d) उपर्युक्त सभी
उत्तर : (d)
व्याख्या: ऐसे पौधे, जीवाणु, कवक व जंतु जिनके जीन, तकनीक की मदद से परिवर्तित किये जा चुके हैं, आनुवंशिक रूपांतरित जीव (जेनेटिकली मोडीफाइड आर्गेनिज़्म – जीएमओ) कहलाते हैं। जीएमओ का व्यवहार स्थानांतरित जीन की प्रकृति, परपोषी पौधे, जंतुओं या जीवाणुओं की प्रकृति व खाद्य जाल पर निर्भर करता है। जीएम पौधे का उपयोग कई प्रकार से लाभदायक है। आनुवंशिक रूपांतरण द्वारा-
• अजैव प्रतिबलों (ठंडा, सूखा, लवण, ताप) के प्रति अधिक सहिष्णु फसलों का निर्माण।
• रासायनिक पीड़कनाशकों पर कम निर्भरता करना (पीड़कनाशी – प्रतिरोधी फसल) ।
• कटाई पश्चात् होने वाले नुकसानों को कम करने में सहायक ।
• पौधे द्वारा खनिज उपयोग क्षमता में वृद्धि ( यह शीघ्र मृदा उर्वरता समापन को रोकता है)।
• खाद्य पदार्थों के पोषणिक स्तर में वृद्धि उदाहरणार्थ- विटामिन ए से समृद्ध धान । उपर्युक्त उपयोगों के साथ-साथ जीएम का उपयोग तदनुकुल पौधों के निर्माण में सहायक है, जिनसे वैकल्पिक संसाधनों के रूप में उद्योगों में वसा, ईंधन व भेषजीय पदार्थों की आपूर्ति की जाती है।
2. कपास के कीट -रोधी पौधे आनुवंशिक इंजीनियरिंग द्वारा एक जीन को निर्विष्ट कर निर्मित किये गए हैं, जो लिया गया है-
(a) विषाणु से
(b) जीवाणु से
(c) फफूंद से
(d) कीट से
उत्तर : (b)
व्याख्या : आनुवंशिक इंजीनियरिंग का प्रयोग कर बीटी (Bt) जो एक जीवाणु है, जिसे बैसिलस थुरीनजिएंसीस (Bt) से प्राप्त करते हैं से कपास के कीट -रोधी पौधे तैयार किये जाते हैं। बीटी जीवविष (टॉक्सिन) जीन जीवाणु से क्लोनिकृत होकर पौधों में अभिव्यक्त होकर कीटों (पीड़कों) के प्रति प्रतिरोधकता पैदा करता है, जिससे कीटनाशकों के उपयोग की आवश्यकता नहीं रह जाती है।
3. गोल्डन चावल एक प्रचुरतम स्रोत है-
(a) विटामिन A
(b) विटामिन B
(c) विटामिन D
(d) विटामिन E
उत्तर : (a)
व्याख्या : जेनेटिक इंजीनियरिंग का प्रयोग कर खाद्य पदार्थों के पोषण स्तर में वृद्धि की जाती है। इसका सर्वोत्तम उदाहरण है गोल्डन राइस, जिसमें विटामिन A प्रचुर मात्रा में पाई जाती है।
4. बी. थूरीनजिएंसीस (Bt) जीवाणु अपनी वृद्धि के विशेष अवस्था में विषाक्त कीटनाशक प्रोटीन का निर्माण करता है। यह विषाक्त प्रोटीन Bt जीवाणु को क्यों नहीं मारता?
(a) जीवाणु के प्रति प्रतिरोधी है।
(b) विष, अपरिपक्व होते हैं।
(c) विष, निष्क्रिय होते हैं।
(d) विष, जीवाणु की विशेष थैली में मिलता है।
उत्तर : (c)
व्याख्या : बीटी ( बैसीलस थूरीनजिएंसीस) की कुछ नस्लें ऐसे प्रोटीन का निर्माण करती हैं जो विशिष्ट कीटों जैसे कि लीथीडोप्टेशन (तंबाकू की कलिका कीड़ा, सैनिक कीड़ा), कोलियोप्टेरान (भृंग) व डीप्टेरॉन (मक्खी, मच्छर) को मारने में सहायक है।
• बी. भूरीनजिएसीस अपनी वृद्धि की विशेष अवस्था में कुछ प्रोटीन रवों का निर्माण करती है। इन रवों में विषाक्त कीटनाशक प्रोटीन होता है। वास्तव में बीटी जीव – विष प्रोटीन, जीवाणु में निष्क्रिय रूप में होता है, ज्योंहि कीट इस निष्क्रिय जीव विष को खाता है, इसके रवे आँत में क्षारीय पीएच के कारण घुलनशील होकर सक्रिय रूप में परिवर्तित हो जाते हैं। सक्रिय जीव विष मध्य औत के उपकलीय कोशिकाओं की सतह से बँधकर उसमें छिद्रों का निर्माण करते हैं, जिस कारण से कोशिकाएँ फूलकर फट जाती हैं और परिणामस्वरूप कीट की मृत्यु हो जाती है।
5. चिकित्सा के क्षेत्र में जैव प्रौद्योगिकी के उपयोग के संबंध नीचे दिये गए कथनों पर विचार कीजिये-
1. इस प्रौद्योगिकी द्वारा विकसित औषधियों का शरीर पर अवांछित प्रतिरक्षात्मक (Unwanted Immunological) प्रभाव नहीं पड़ता है।
2. इस प्रौद्योगिकी के द्वारा मानव शरीर में उत्पन्न होने वाले सारे रोगों का उपचार संभव है।
उपर्युक्त में से कौन – सा /से कथन सत्य है / हैं?
(a) केवल 1
(b) केवल 2
(c) 1 और 2 दोनों
(d) न तो 1 और न ही 2
उत्तर : (a)
व्याख्या: केवल पहला कथन सत्य है। जैव प्रौद्योगिकी ने स्वास्थ्य सुरक्षा के क्षेत्र में अत्यधिक प्रभाव डाला है, क्योंकि इसके द्वारा उत्पन्न सुरक्षित व अत्यधिक प्रभावी चिकित्सीय औषधियों का उत्पादन अधिक मात्रा में संभव है । पुनर्योगज चिकित्सीय औषधियों का अवांछित प्रतिरक्षात्मक (Unwanted Immunological) प्रभाव नहीं पड़ता है जबकि ऐसा देखा गया है कि उपर्युक्त उत्पाद जो अमानवीय स्रोतों से विलगित किये गए हैं, वे अवांछित प्रतिरक्षात्मक प्रभाव डालते हैं।
6. निम्नलिखित कथनों पर विचार कीजिये-
1. मधुमेह रोग से पीड़ित व्यक्ति को इसके नियंत्रण के लिये इंसुलिन ना पड़ता है।
2. मधुमेह रोग में उपयोग लाया जाने वाला इंसुलिन जानवरों व सुअरों को मारकर उनके अग्न्याशय से प्राप्त किया जाता था।
3. वर्तमान में जैव प्रौद्योगिकी की सहायता से जीवाणुओं द्वारा इंसुलिन का निर्माण किया जाता है।
उपर्युक्त में से कौन – सा /से कथन सत्य है / हैं?
(a) केवल 1 और 2
(b) केवल 2
(c) केवल 2 और 3
(d) 1, 2 और 3
उत्तर : (d)
व्याख्या : उपर्युक्त तीनों कथन सत्य हैं।
7. आनुवंशिक विकार के साथ पैदा हुए बच्चे के उपचार हेतु जीन चिकित्सा के अंतर्गत जीन को व्यक्ति के शरीर के किस भाग में प्रवेश कराया जाता है?
1. माइटोकांड्रिया
2. कोशिका
3. ऊतक
नीचे दिये गए कूट का प्रयोग कर सही उत्तर चुनिये-
(a) केवल 1 और 2
(b) केवल 2 और 3
(c) केवल 1 और 3
(d) उपर्युक्त सभी।
उत्तर : (b)
व्याख्या: यदि कोई बच्चा आनुवंशिक रोग के साथ पैदा हुआ है तो इस रोग के उपचार हेतु जीन चिकित्सा ही एकमात्र उपाय है। जीन चिकित्सा में उन विधियों का सहयोग लेते हैं जिनके द्वारा किसी बच्चे या भ्रूण में चिह्नित किये गए जीन दोषों का सुधार किया जाता है। इसमें रोगों के उपचार हेतु जीनों को व्यक्ति की कोशिकाओं या ऊतकों में प्रवेश कराया जाता है। आनुवंशिक दोष वाली कोशिकाओं के उपचार हेतु सामान्य जीन को व्यक्ति या भ्रूण में स्थानांतरित करते हैं जो निष्क्रिय जीन की क्षतिपूर्ति कर उसके कार्यों को संपन्न करते हैं।
8. निम्नलिखित में से कौन-सा कथन बायोपाइरेसी को परिभाषित करता है?
(a) जैव प्रौद्योगिकी की सहायता से ट्रांसजेनिक एनिमल्स (पराजीनी जंतु ) का निर्माण करना ।
(b) कंपनी या संगठन द्वारा किसी राष्ट्र या उससे संबंधित लोगों से बिना व्यवस्थित अनुमोदन व क्षतिपूरक भुगतान के जैव संसाधनों का उपयोग करना ।
(c) पौधों एवं जंतुओं का अवैध व्यापार करना ।
(d) किसी भी सूक्ष्मजीव, पौधे एवं जंतुओं के आनुवंशिक गुणों में परिवर्तन करना ।
उत्तर : (b)
व्याख्या : मल्टीनेशनल कंपनियों व दूसरे संगठनों द्वारा किसी राष्ट्र या उससे संबंधित लोगों से बिना व्यवस्थित अनुमोदन व क्षतिपूरक भुगतान के जैव संसाधनों का उपयोग करना बायोपाइरेसी कहलाता है।
9. निम्नलिखित कथनों पर विचार कीजिये-
1. उन्नत जीवों जैस- सूक्ष्मजीवों या शुद्ध एंजाइम के रूप में सर्वोत्तम उत्प्रेरक का निर्माण करना।
2. अनुप्रवाह प्रक्रमण तकनीक का प्रोटीन / कार्बनिक यौगिक के शुद्धीकरण में उपयोग करना ।
उपर्युक्त कथनों में से कौन-सा / से जैव प्रौद्योगिकी (Biotechnology) के अनुसंधान क्षेत्र है/हैं?
(a) केवल 1
(b) केवल 2
(c) 1 एवं 2 दोनों
(d) न तो 1 और न ही 2
उत्तर : (c)
व्याख्या : जैव-प्रौद्योगिकी वह तकनीक है, जिसमें जीवधारियों या जीवधारियों से प्राप्त होने वाले पदार्थों का उपयोग करके उत्पादों का निर्माण या उनमें सुधार का अध्ययन किया जाता है।
• इसके अंतर्गत स्वास्थ्य, चिकित्सा, कृषि, पशुपालन, उद्योग और पर्यावरण आदि जैसे अनेक क्षेत्र सम्मिलित हैं। हैं। जैव प्रौद्योगिकी के तीन विवेचनात्मक अनुसंधान क्षेत्र हैं-
• उन्नत जीवों जैसे- सूक्ष्मजीवों या शुद्ध एंजाइम के रूप में सर्वोत्तम उत्प्रेरक का निर्माण करना।
• उत्प्रेरक के कार्य हेतु अभियांत्रिकी द्वारा सर्वोत्तम परिस्थितियों का निर्माण करना।
• अनुप्रवाह प्रक्रमण (Downstream Processing) तकनीक का प्रोटीन / कार्बनिक यौगिक के शुद्धीकरण में उपयोग करना। इसलिये, दोनों कथन सही हैं।
अतः विकल्प (c) सही उत्तर है।
10. निम्नलिखित कथनों पर विचार कीजिये-
1. अजैव प्रतिबलों के प्रति अधिक सहिष्णु फसलों का निर्माण
2. पौधों द्वारा खनिज उपयोग क्षमता में वृद्धि
3. फसलों की कम लागत
4. फसलों की जैव-विविधता में वृद्धि
उपर्युक्त कथनों में से जीएम फसलों (GM-Crops) के लाभों के संदर्भ में कौन-सा/से सही है/हैं?
(a) केवल 1 और 3
(b) केवल 2
(c) केवल 1 और 2
(d) कंवल 1, 3 और 4
उत्तर : (c)
व्याख्या : जीएम फसल उन फसलों को कहा जाता है जिसमें जीन को वैज्ञानिक तरीके से रूपांतरित किया जाता है। जीएम फसलों का उपयोग कई प्रकार से लाभदायक है-
• यह बीज साधारण बीज से कहीं अधिक उत्पादकता प्रदान करता है।
• यह सूखा-रोधी और बाढ़-रोधी होने के साथ कीट प्रतिरोधी भी है, जो अधिक सहिष्णु फसलों के निर्माण में सहायक है।
• रासायनिक पीड़कनाशियों पर कम निर्भरता
• पौधों / फसलों द्वारा खनिज उपयोग क्षमता में वृद्धि। (यह शीघ्र मृदा उर्वरता समापन को रोकता है।)
• खाद्य पदार्थों के पोषण स्तर में वृद्धि
• जबकि भारत में इन फसलों का विरोध होता है क्योंकि इसकी लागत अधिक होती है तथा इन्हें पर्यावरण व जैव-विविधता के लिये हानिकारक माना जाता है। अतः केवल कथन 1 और कथन 2 सही हैं।
अत: विकल्प (c) सही उत्तर है।
11. इंसुलिन के संदर्भ में निम्नलिखित कथनों पर विचार कीजिये –
1. इंसुलिन दो छोटी पॉलीपेप्टाइड श्रृंखलाओं का बना होता है।
2. इंसुलिन रक्त में शर्करा की मात्रा नियंत्रित करता है।
3. मानव में इंसुलिन की कार्यक्षमता में कमी होने पर मधुमेह रोग हो जाता है।
उपर्युक्त कथनों में से कौन-सा/से सही है/हैं?
(a) केवल 3
(b) केवल 2 और 3
(c) केवल 1 और 2
(d) 1, 2 और 3
उत्तर : (d)
व्याख्या : इंसुलिन दो छोटी पॉलीपेप्टाइड श्रृंखलाओं का बना होता है, श्रृंखला – A और श्रृंखला-B जो आपस में डाइसल्फाइड बंधों द्वारा जुड़ी
• इंसुलिन एक प्रकार का हार्मोन है जिसका निर्माण अग्न्याशय में होता है।
• जब मानव शरीर में अग्न्याशय (Pancreas) द्वारा इंसुलिन नामक हॉर्मोन का स्रावण कम हो जाता है अथवा इंसुलिन की कार्यक्षमता में कमी आ जाती है, तो मधुमेह (Diabetes) रोग हो जाता है।
• इंसुलिन रक्त में शर्करा की मात्रा को नियंत्रित करता है । अतः तीनों कथन सही हैं।
अत: विकल्प (d) सही उत्तर है।
12. निम्नलिखित में से कौन-सी पहली ट्रांसजेनिक फसल थी?
1. सूत
2. मटर
3. तंबाकू
नीचे दिये गए कूट में से सही उत्तर चुनिये-
(a) केवल 1
(b) केवल 2 और 3
(c) केवल 3
(d) 1, 2 और 3
उत्तर : (c)
व्याख्या : जेनेटिक इंजीनियरिंग के जरिए किसी भी जीव या पौधों के जीन को दूसरे पौधों में प्रवेश कराकर नई फसल प्रजाति विकसित की जाती है। 1982 में तंबाकू के पौधे में इसका पहला प्रयोग किया गया था। जबकि फ्राँस और अमेरिका में 1986 में पहली बार इसका फील्ड ट्रायल दिया गया था। इस प्रकार, तंबाकू पहली ट्रांसजेनिक फसल थी । अत: विकल्प (c) सही उत्तर हैं।

13. जीव और समष्टियाँ

1. पारिस्थितिकी के अंतर्गत जीवीय संगठन के कौन-से स्तर सम्मिलित होते हैं?
1. जीव
2. समष्टियाँ
3. समुदाय
4. जीवोम
कूट :
(a) केवल 1 और 2
(b) केवल 2, 3 और 4
(c) केवल 3 और 4
(d) उपर्युक्त सभी।
उत्तर : (d)
व्याख्या: पारिस्थितिकी के अंतर्गत जीवों के बीच या जीवीय तथा भौतिक (अजीवीय/अवायोटिक) पर्यावरण के बीच होने वाली पारस्परिक क्रियाओं का अध्ययन किया जाता है।
• मूलरूप से पारिस्थितिकी जीवीय संगठन के चार स्तरों से संबंधित है- जीव, समष्टियाँ, समुदाय और जीवोम |
2. निम्नलिखित में से कौन-से जीवों के जीवन को प्रभावित करने वाले महत्त्वपूर्ण अजैव कारक हैं?
1. तापमान
2. जल
3. प्रकाश
4. मृदा
कूट:
(a) केवल 1 और 2
(b) केवल 2 और 3
(c) केवल 1 और 3
(d) उपर्युक्त सभी।
उत्तर : (d)
व्याख्या: जीवों के जीवन को प्रभावित करने वाले महत्त्वपूर्ण कारक तापमान, जल, प्रकाश और मृदा हैं।
• तापमान: तापमान परिस्थितिक रूप से सबसे ज्यादा प्रासंगिक पर्यावरणीय कारक है। पृथ्वी पर औसत तापमान ऋतु के अनुसार बदलता रहता है। भूमध्य रेखा से ध्रुवों की ओर और मैदानों से पर्वत शिखरों की ओर जाने पर तापमान उत्तरोतर घटता जाता है।
• जल : तापमान के बाद, जीवों के जीवन को प्रभावित करने वाला महत्त्वपूर्ण कारक जल है। क्योंकि जल के बिना जीवन असंभव है। जलीय जीवों के लिये जल की गुणवत्ता महत्त्वपूर्ण होती है। लवण की सांद्रता अंतः स्थलीय जल में 5 से कम, समुद्र में 30-35 और कुछ अतिलवणीय लैगूनों में 100 से अधिक होती है। कुछ जीव लवणता की व्यापक परास के प्रति सहनशील होते हैं, लेकिन अन्य कम परास में सीमित होते हैं। बहुत से अलवण जल प्राणी समुद्र के पानी में और समुद्री प्राणी अलवण में लंबे समय तक नहीं रह सकते, क्योंकि उन्हें परासरणी समस्याओं का सामना करना पड़ता है।
• प्रकाश : पौधे प्रकाश संश्लेषण द्वारा खाद्य उत्पन्न करते हैं। यह एक ऐसी प्रक्रिया है जो ऊर्जा के स्रोत के रूप में धूप उपलब्ध होने पर संभव है। इसलिये हम जीवधारियों के लिये विशेषरूप से स्वपोषियों के लिये, प्रकाश महत्त्वपूर्ण है।
• मृदा : विभिन्न स्थानों में मृदा की प्रकृति और गुण भिन्न-भिन्न होते हैं। मृदा की विभिन्न विशेषताएँ जैसे कि मृदा का संघटक, कण-साइज ही काफी हद तक किसी क्षेत्र की वनस्पति का निर्धारण करते हैं।
3. जीव का कोई एक ऐसा गुण ( आकारिकीय, कार्यिकीय, व्यावहारिक) जो उसे अपने आवास में जीवित बने रहने और जनन करने के योग्य बनाता है, कहलाता है-
(a) निलंबन
(b) अनुकूलन
(c) संरूपण
(d) उत्प्रवासन
उत्तर : (b)
व्याख्या : अनुकूलन जीव का एक ऐसा गुण ( आकारिकीय, कार्यिकीय, व्यावहारिक) है, जो इसे अपने आवास में जीवित बने रहने के लिये और जनन करने के योग्य बनाता है। अनेक अनुकूलन लंबे समय की विकास यात्रा के बाद विकसित हुए हैं और आनुवंशिकतः स्थिर हो गए हैं।
4. निम्नलिखित कथनों पर विचार कीजिये-
1. मरुस्थलीय पौधों के प्रकाश संश्लेषी मार्ग विशेष प्रकार के होते हैं, जिसके कारण वे अपने रंध्र दिन के समय बंद रखते हैं।
2. मरुस्थलीय पौधे नागफनी में प्रकाश संश्लेषण का कार्य चपटे तनों द्वारा होता है।
उपर्युक्त में से कौन-सा/से कथन सत्य है/हैं?
(a) केवल 1
(b) केवल 2
(c) 1 और 2 दोनों
(d) न तो 1 और न ही 2
उत्तर : (c)
व्याख्या : उपर्युक्त दोनों कथन सत्य हैं।
• अनेक मरुस्थलीय पौधों की पत्तियों की सतह पर मोटी उपत्वचा (क्यूटिकल) होती है और उनके रंध्र (स्टोमैटा) गहरे गर्त में व्यवस्थित होते हैं, ताकि वाष्पोत्सर्जन द्वारा जल की न्यूनतम हानि हो सके। उनके प्रकाश संश्लेषी (सीएएम) मार्ग भी विशेष प्रकार के होते हैं, जिसके कारण वे अपने रंध्र दिन के समय बंद रख सकते हैं।
• मरुस्थलीय पादपों नागफनी (ओपंशिया) कैक्टस आदि में पत्तियाँ नहीं होती, बल्कि वे काँटे के रूप में रूपांतरित हो जाती हैं और प्रकाश संश्लेषण का कार्य चपटे तनों द्वारा होता है।
5. परभक्षण के संदर्भ में नीचे दिये गए कथनों पर विचार कीजिये –
1. परभक्षी, पादपों द्वारा स्थिर की गई ऊर्जा उच्चतर पोषी स्तरों को स्थानांतरित करते हैं।
2. परभक्षी, शिकार समष्टि को नियंत्रित रखते हैं।
3. परभक्षी, किसी समुदाय में जातियों की विविधता को भी बनाए रखने में मदद करते हैं।
उपर्युक्त में से कौन-सा/से कथन सत्य है/हैं?
(a) केवल 1 और 2
(b) केवल 2
(c) केवल 2 और 3
(d) उपर्युक्त सभी।
उत्तर : (d)
व्याख्या : उपर्युक्त सभी कथन सत्य हैं।
• परभक्षण प्रकृति का ऐसा तरीका है जिसमें पादपों द्वारा स्थिर की गई ऊर्जा उच्चतर पोषी स्तरों को स्थानांतरित होती है। पौधों को खाने वाले प्राणी या प्राणियों को खाने वाले दूसरे प्राणियों को परभक्षी कहा जाता है। हालाँकि इन्हें शाकाहारी व माँसाहारी के रूप में भी वर्गीकृत किया जाता है।
• परभक्षी पारितंत्र में एक अन्य महत्त्वपूर्ण भूमिका निभाते हैं। ये शिकार समष्टि को नियंत्रित रखते हैं। अगर परभक्षी नहीं होते तो शिकार जातियों का समष्टि घनत्व बहुत ज़्यादा हो जाता और पारितंत्र में अस्थिरता आ जाती।
• जब किसी भौगोलिक क्षेत्र में कुछ विदेशज जातियाँ लाई जाती हैं तो. वे आक्रामक हो जाती हैं और तेज़ी से फैलने लगती हैं, क्योंकि आक्रांत भूमि पर उसके प्राकृतिक परभक्षी नहीं होते हैं।
• परभक्षी, स्पर्धी शिकार जातियों के बीच स्पर्धा की तीव्रता कम करके किसी समुदाय में जातियों की विविधता बनाए रखने में भी सहायता करते हैं।
6. लाइकेन, जो पारिस्थितिक अनुक्रम को प्रारंभ करने में सक्षम हैं, वास्तव में यह सहजीवी साहचर्य है-
(a) शैवाल और जीवाणु
(b) शैवाल और कवक
(c) जीवाणु और कवक
(d) कवक और फंजाई
उत्तर : (b)
व्याख्या: कवक और प्रकाश संश्लेषी शैवाल या सायनोबैक्टीरिया के बीच घनिष्ठ सहोपकारी (Mutualistic) संबंध का उदाहरण लाइकेन है।
• शैवाल द्वारा कवक को भोजन की आपूर्ति की जाती है। बदले में कवक द्वारा शैवाल को सुरक्षा, जल, नाइट्रोजनी पदार्थ एवं खनिज लवण प्रदान किये जाते हैं।

14. पारितंत्र

1. पारिस्थितिक तंत्र के संबंध में निम्नलिखित कथनों पर विचार कीजिये-
1. पृथ्वी पर पाए जाने वाले सभी जीवधारी आपस में तथा अपने आस-पास के भौतिक पर्यावरण के साथ परस्पर क्रिया करते हैं।
2. किसी पारितंत्र में विभिन्न स्तरों पर विभिन्न प्रजातियों के ऊर्ध्वाधर वितरण को स्तर विन्यास कहते हैं।
उपर्युक्त में से कौन – सा /से कथन सत्य है/हैं?
(a) केवल 1
(b) केवल 2
(c) 1 और 2 दोनों
(d) न तो 1 और न ही 2
उत्तर : (c)
व्याख्या : उपर्युक्त दोनों कथन सत्य हैं।
• पारितंत्र प्रकृति की एक क्रियाशील इकाई है, जहाँ पर जीवधारी आपस में तथा आस-पास के भौतिक पर्यावरण के साथ परस्पर क्रिया करते हैं। पारितंत्र का आकार एक छोटे से तालाब से लेकर एक विशाल जंगल या महासागर तक हो सकता है। जैविक और अजैविक घटकों की परस्पर क्रियाओं के फलस्वरूप एक भौतिक संरचना विकसित होती है, जो प्रत्येक प्रकार के पारितंत्र की विशिष्टता है।
• एक पारितंत्र की पादप एवं प्राणी प्रजातियों की पहचान एवं गणना इसकी प्रजातियों के संघटन (कंपोजीशन ) को प्रकट करता है। विभिन्न स्तरों पर विभिन्न प्रजातियों के ऊर्ध्वाधर वितरण को स्तर विन्यास कहते हैं। उदाहरण: एक जंगल में वृक्ष सर्वोपरि ऊर्ध्वाधर स्तर, झाड़ियाँ द्वितीयक स्तर तथा जड़ी-बूटियाँ एवं घास निचले ( धरातलीय) स्तर पर निवास करती हैं।
2. निम्नलिखित में से कौन-सा कथन पारिस्थितिक तंत्र के प्राथमिक उत्पादन को परिभाषित करता है?
(a) प्रकाश संश्लेषण के दौरान पादपों द्वारा एक निश्चित समयावधि में प्रति इकाई क्षेत्र में उत्पन्न की गई कार्बनिक सामग्री की मात्रा।
(b) प्रकाश संश्लेषण के द्वारा उत्पन्न की गई कार्बनिक सामग्री एवं दैनिक गतिविधियों में खर्च की गई ऊर्जा का अनुपात ।
(c) पादपों द्वारा प्रकाश संश्लेषण के लिये पृथ्वी से ली गई कार्बनिक पदार्थों की मात्रा |
(d) किसी निश्चित क्षेत्र के जीवधारियों द्वारा जैव मात्रा का उपभोग ।
उत्तर : (a)
व्याख्या : प्राथमिक उत्पादन प्रकाश संश्लेषण के दौरान पादपों द्वारा एक निश्चित समयावधि में प्रति इकाई क्षेत्र में उत्पन्न की गई जैव मात्रा या कार्बनिक सामग्री है। जैव मात्रा की उत्पादन की दर को उत्पादकता कहते हैं। इसके द्वारा ही विभिन्न पारितंत्रों की उत्पादकता की तुलना की जा सकती है।
3. किसी भी पारितंत्र में निम्नलिखित में से किनको अपघटक जीव कहते हैं?
1. केंचुआ
2. जीवाणु
3. विषाणु
4. कवक
नीचे दिये गए कूट का प्रयोग कर सही उत्तर चुनिये-
(a) केवल 1 और 2
(b) केवल 2, 3 और 4
(c) केवल 3 और 4
(d) केवल 1, 2 और 4
उत्तर : (d)
व्याख्या : अपघटक वे जीव होते हैं जो जटिल कार्बनिक यौगिकों को अकार्बनिक तत्त्वों जैसे- कार्बन डाइऑक्साइड, जल एवं पोषकों में खंडित करने में सहायता करते हैं। इस प्रक्रिया को अपघटन कहते हैं। केंचुआ, जीवाणु तथा कवक प्रमुख प्राकृतिक अपघटक हैं।
4. निम्नलिखित कथनों पर विचार कीजिये-
1. अपघटन की प्रक्रिया में ऑक्सीजन की आवश्यकता नहीं होती है।
2. अपरद पदार्थों में यदि नाइट्रोजन तथा जलविलेय जैसे तत्त्व हों तो अपरदन की प्रक्रिया तेज़ होती है।
3. निम्न ताप एवं अवायुजीवन अपघटन की गति को धीमा करते हैं।
उपर्युक्त में से कौन – सा/से कथन सत्य है/हैं?
(a) केवल 1
(b) केवल 2 और 3
(c) केवल 1 और 3
(d) 1, 2 और 3
उत्तर : (b)
व्याख्या: पहला कथन असत्य है। अपघटन की प्रक्रिया में ऑक्सीजन की आवश्यकता होती है।
• दूसरा व तीसरा कथन सत्य हैं। अपघटन की दर जलवायुवीय घटकों तथा अपरद (पादपों के मृद अवशेष जैसे- पत्तियाँ, छाल, फूल तथा प्राणियों के मृत अवशेष) के रासायनिक संघटनों द्वारा निर्धारित होती है। यदि अपरद में नाइट्रोजन तथा जलविलेय तत्त्वों (जैसे- चीनी आदि) की मात्रा भरपूर होती है तब अपघटन की दर तीव्र होती है।
• ताप एवं मृदा की नमी बहुत ही महत्त्वपूर्ण जलवायुवीय घटक हैं जो मृदा के सूक्ष्मजीवों की क्रियाओं द्वारा अपघटन की गति को नियमित करते हैं। गरम एवं आर्द्र पर्यावरण में अपघटन की गति तेज़ होती है, जबकि निम्न ताप एवं अवायुजीवन अपघटन की गति को धीमा करते हैं जिसके परिणामस्वरूप कार्बनिक पदार्थों का भंडार जमा हो जाता है।
5. निम्नलिखित कथनों पर विचार कीजिये-
1. पारिस्थितिक तंत्र में ऊर्जा का प्रवाह एकदिशीय होता है।
2. पारिस्थितिक तंत्र में पादप एवं जंतु दोनों को उत्पादक कहा जाता है।
3. कवक एवं जीवाणु अपरद खाद्य श्रृंखला के महत्त्वपूर्ण घटक हैं।
उपर्युक्त में से कौन-से कथन सत्य हैं ?
(a) केवल 1 और 2
(b) केवल 2 और 3
(c) केवल 1 और 3
(d) 1, 2 और 3
उत्तर : (c)
व्याख्या: सभी जीव आहार के लिये प्रत्यक्ष या अप्रत्यक्ष रूप से उत्पादकों पर निर्भर रहते हैं। अतः सूर्य से उत्पादकों की ओर और फिर उत्पादकों से उपभोक्ता की ओर ऊर्जा का प्रवाह एकदिशीय होता है। अत: पहला कथन सत्य है।
• दूसरा कथन असत्य है। पारिस्थितिक तंत्र की शब्दावली में हरे पादप को उत्पादक कहा जाता है।
• मृत कार्बनिक सामग्री से प्रारंभ होने वाली खाद्य श्रृंखला को अपरद खाद्य श्रृंखला के नाम से जाना जाता है। यह खाद्य श्रृंखला अपघटकों से बनी होती है जो कि मुख्यतः कवक एवं जीवाणु के रूप में परपोषित जीव होते हैं। अपघटक पाचक एंजाइम्स स्रावित करते हैं जो मृत जीवों तथा व्यर्थ सामग्री को साधारण अकार्बनिक पदार्थों में तोड़ डालते हैं, जो बाद में उन्हीं के द्वारा अवशोषित कर लिये जाते हैं।
6. पारिस्थितिक तंत्र में खाद्य श्रृंखला के अंतर्गत निम्न पोषण स्तर से उच्च पोषण स्तर में ऊर्जा का कितना प्रतिशत (%) भाग स्थानांतरित होता है ?
(a) 5%
(b) 10%
(c) 90%
(d) 100%
उत्तर : (b)
व्याख्या: खाद्य श्रृंखला में प्रत्येक निम्न पोषण स्तर से उच्चतर पोषण स्तर में केवल 10 प्रतिशत ऊर्जा का ही स्थानांतरण होता है। सबसे महत्त्वपूर्ण बात यह है कि उत्तरोत्तर पोषण स्तरों पर ऊर्जा की मात्रा घटती जाती है। खाद्य श्रृंखला के प्रत्येक पोषण स्तर पर जीव अपनी ऊर्जा की आवश्यकता के लिये निम्न पोषण स्तर पर निर्भर रहता है।
7. निम्नलिखित में से किस पारिस्थितिक तंत्र की उत्पादकता सबसे कम है?
(a) मैंग्रोव वन
(b) घास स्थल
(c) झील
(d) समुद्र
उत्तर : (d)
व्याख्या : दिये गए विकल्पों में सबसे कम उत्पादकता समुद्रों की होती है। संपूर्ण जीवमंडल की वार्षिक कुल प्राथमिक उत्पादकता का भार 170 बिलियन टन आँका गया है। यद्यपि पृथ्वी के धरातल का लगभग 70 प्रतिशत भाग समुद्रों से ढका हुआ है, इसके बावजूद इनकी उत्पादकता केवल 55 बिलियन टन है। शेष मात्रा भूमि पर पैदा होती है।
8. एक खाद्य श्रृंखला में निम्नलिखित में से सर्वाधिक संख्या किसकी होती है?
(a) उत्पादक
(b) प्राथमिक उपभोक्ता
(c) द्वितीयक उपभोक्ता
(d) अपघटक
उत्तर : (a)
व्याख्या: किसी भी खाद्य श्रृंखला में उत्पादक सर्वाधिक संख्या में होते हैं।
9. पारिस्थितिक पिरामिड के संबंध में कौन-सा कथन असत्य है ?
(a) एक समय पर, एक ही पारिस्थितिक तंत्र में एक कोई भी जीव केवल एक ही पोषण रीति में अधिष्ठित हो सकता है।
(b) निम्न पोषण स्तर में ऊर्जा की मात्रा ऊपरी पोषण स्तर से अधिक होती है।
(c) ऊर्जा पिरामिड सदैव सीधा होता है, कभी उल्टा नहीं हो सकता है।
(d) समुद्र के जैव भार (मात्रा) के पिरामिड उल्टे होते हैं।
उत्तर : (a)
व्याख्या: कथन (a) असत्य है। एक प्रदत्त प्रजाति, एक ही समय पर एक ही पारिस्थितिक तंत्र में एक से अधिक पोषण रीतियों में अधिष्ठित हो सकती है। उदाहरण के लिये एक गौरेया जब बीज, फल व मटर खाती है वह प्राथमिक उपभोक्ता है किंतु जब वह कीटों एवं केंचुओं को खाती है, तब वह द्वितीयक उपभोक्ता है। इसी प्रकार मानव को भी प्राथमिक, द्वितीयक एवं तृतीयक तीनों पोषण रीतियों में शामिल कर सकते हैं।
10. किसी भी पारितंत्र के सुनिश्चित क्षेत्र की प्रजाति संरचना में उचित रूप से आकलित परिवर्तन को कहते हैं-
(a) इकोटोन
(b) पारिस्थितिक क्रमक
(c) पारिस्थितिक अनुक्रमण
(d) पारिस्थितिक निशे (Niche)
उत्तर : (c)
व्याख्या: सभी समुदायों का महत्त्वपूर्ण लक्षण पर्यावरण के बदलते स्वरूप के साथ इसके संगठन एवं संरचना में निरंतर परिवर्तन होते रहना है। यह परिवर्तन क्रमबद्ध और भौतिक पर्यावरण के परिवर्तन के समानांतर होता है। अंततः यह परिवर्तन एक समुदाय को गठित करता है, जो कि पर्यावरण से संतुलन के नज़दीक है और इसे चरम समुदाय कहा जाता है। एक सुनिश्चित क्षेत्र की प्रजाति संरचना में उचित रूप से आकलित परिवर्तन को पारिस्थितिक अनुक्रमण कहते हैं।
11. पारिस्थितिक तंत्र में निम्नलिखित में से कौन-सा जैव भार खाद्य पिरामिड उल्टा हो जाता है?
(a) वन
(b) घास स्थल
(c) शुष्क स्थल
(d) समुद्र
उत्तर : (d)
व्याख्या: समुद्र में जैव भार के खाद्य पिरामिड प्रायः उल्टे होते हैं, क्योंकि मछलियों की जैव मात्रा पादप प्लवकों की जैव मात्रा से बहुत अधिक होती है।
12. पारिस्थितिक अनुक्रमण के संबंध में नीचे दिये गए कथनों पर विचार कीजिये-
1. अनुक्रमण की प्रक्रिया वहाँ शुरू होती है, जहाँ सजीव नहीं होते हैं।
2. अनुक्रमण के दौरान किसी क्षेत्र में कुछ प्रजातियाँ विलुप्त भी हो सकती हैं।
3. पारिस्थितिक अनुक्रमण का वितरण वानस्पतिक परिवर्तन पर केंद्रित होता है।
उपर्युक्त में से कौन-सा/से कथन सत्य है/हैं?
(a) केवल 1
(b) केवल 2 और 3
(c) केवल 3
(d) 1, 2 और 3
उत्तर : (d)
व्याख्या: उपर्युक्त सभी कथन सत्य हैं।
• अनुक्रमण एक प्रक्रिया है, जो वहाँ शुरू होती है, जहाँ कोई सजीव नहीं होता अथवा कोई ऐसा क्षेत्र जहाँ कभी कोई सजीव नहीं रहा हो, उदाहरण के लिये नग्न पत्थर या फिर ऐसा क्षेत्र जहाँ के सभी जीव किसी प्रकार से नष्ट (लुप्त ) हो गए।
• अनुक्रमण की प्रक्रिया के दौरान कुछ प्रजातियाँ एक क्षेत्र में नई बस्ती बसा लेती हैं और इनकी संख्या अनगिनत हो जाती है, जबकि दूसरी प्रजातियों की संख्या घटती चली जाती है और यहाँ तक कि विलुप्त हो जाती है। संसार के निर्वतमान समुदाय धरती पर जीवोत्पत्ति के पश्चात् लाखों वर्षों के अनुक्रमण के फलस्वरूप उत्पन्न हुए हैं। वास्तव अनुक्रमण एवं विकास उस समय समानांतर प्रक्रियाएँ थीं।
• साधारणतया पारिस्थितिक अनुक्रमण का वितरण वानस्पतिक परिवर्तन पर केंद्रित होता है। जबकि बाद में ये वानस्पतिक परिवर्तन, विभिन्न प्रकार के जानवरों के खाद्य और शरण स्थल पर प्रभाव डालते हैं।
13. पारिस्थितिक तंत्र में तत्त्वों के चक्रण को क्या कहते हैं?
(a) रासायनिक चक्र
(b) जैव – भूरसायन चक्र
(c) भूवैज्ञानिक चक्र
(d) भूरासायनिक चक्र
उत्तर : (b)
व्याख्या : किसी भी पारितंत्र में जीवों की लगातर वृद्धि, प्रजनन एवं विभिन्न कायिक क्रियाओं को संपन्न करने के लिये लगातार पोषकों के संभरण की आवश्यकता होती है। मृदा में विद्यमान पोषकों की मात्रा, जैसे- कार्बन, नाइट्रोजन, फॉस्फोरस, कैल्शियम आदि को स्थायी अवस्था के रूप में संदर्भित किया जाता है। यह विभिन्न प्रकार के पारितंत्र में भिन्न होती है और साथ ही मौसम पर आधारित होती है। एक पारितंत्र में विभिन्न घटकों के माध्यम से पोषक तत्त्वों की गतिशीलता को पोषक चक्र या जैव-भूरसायन चक्र कहते हैं। पारितंत्र से पोषक कभी समाप्त नहीं होते हैं बल्कि ये बार-बार पुनः चक्रित होते रहते हैं एवं अंनत काल तक चलते रहते हैं।
14. पारिस्थितिक तंत्र के अंतर्गत निम्नलिखित में से कौन-से पोषक तत्त्वों के भंडार वायुमंडल में विद्यमान रहते हैं ?
1. कार्बन
2. नाइट्रोजन
3. सल्फर
4. फॉस्फोरस
कूट :
(a) केवल 1 और 2
(b) केवल 1, 2 और 3
(c) केवल 3 और 4
(d) केवल 2, 3 और 4
उत्तर : (a)
व्याख्या: पारिस्थितिक तंत्र में पोषक चक्र या जैव भू-रासायनिक चक्र दो प्रकार के होते हैं- 1. गैसीय, 2. अवसादीय या तलछटी ।
• गैसीय प्रकार के पोषक चक्र, जैसे- नाइट्रोजन, कार्बन के भंडार वायुमंडल में विद्यमान होते हैं तथा अवसादी पोषक तत्त्व, जैसे- सल्फर एवं फॉस्फोरस के भंडार धरती के पटल पर स्थित होते हैं। पर्यावरणीय घटक, जैसे कि मिट्टी, आर्द्रता, पीएच, ताप आदि वायुमंडल में पोषकों के मुक्त होने की दर तय करते हैं।
15. पारितंत्र में फॉस्फोरस चक्र के संबंध पर निम्नलिखित कथनों पर विचार कीजिये –
1. चट्टानें, फॉस्फोरस के सबसे बड़े प्राकृतिक भंडार हैं।
2. पर्यावरण में फॉस्फोरस को श्वसन द्वारा मुक्त किया जाता है।
उपर्युक्त में से कौन-सा/से कथन सत्य है/हैं? –
(a) केवल 1
(b) केवल 2
(c) 1 और 2 दोनों
(d) न तो 1 और न ही 2
उत्तर : (a)
व्याख्या: पहला कथन सत्य है। फॉस्फोरस का प्राकृतिक भंडार चट्टानों में है जो कि फॉस्फेट के रूप में फॉस्फोरस को संचित किये हुए हैं। जब चट्टानों का अपक्षय होता है तो थोड़ी मात्रा में ये फॉस्फेट भूमि के विलयन में घुल जाते हैं एवं उन्हें पादपों की जड़ों द्वारा अवशोषित कर लिया जाता है। शाकाहारी और अन्य जानवर इन तत्त्वों को पादपों से ग्रहण करते हैं। अनेक प्राणियों को अपना कवच, अस्थियाँ एवं दाँत आदि बनाने के लिये इसकी आवश्यकता होती है।
• दूसरा कथन असत्य है। पर्यावरण में फॉस्फोरस को श्वसन द्वारा मुक्त नहीं किया जाता है। जीवों और पर्यावरण के बीच फॉस्फोरस का गैसीय विनिमय बिल्कुल नगण्य होता है।

15. जैव विविधता एवं संरक्षण

1. निम्नलिखित कथनों पर विचार कीजिए-
1. जैव विविधता शब्द सामाजिक जीव- वैज्ञानिक रॉबर्ट मेए द्वारा दिया गया है।
2. आनुवंशिक विविधता, जातीय विविधता और परिस्थितिकीय विविधता, जैव विविधता के आधार हैं।
उपर्युक्त कथनों में से कौन-सा/से सही है/है?
(a) केवल 1
(b) केवल 2
(c) 1 और 2 दोनों
(d) न तो 1 और न ही 2
उत्तर : (b)
व्याख्या: पहला कथन असत्य है। जैव विविधता शब्द सामाजिक जीव- वैज्ञानिक एडवर्ड विलसन द्वारा दिया गया है। रॉबर्ट मेए वैश्विक जातीय विविधता के आकलन से संबंधित हैं।
• दूसरा कथन सत्य है। एडवर्ड विलसन ने जैविक संगठन के प्रत्येक स्तर पर उपस्थित विविधता को दर्शाने के लिये आनुवंशिक विविधता, जातीय विविधता और पारिस्थितिकीय विविधता को प्रचलित किया।
♦ आनुवंशिक विविधता : इस विविधता के अंतर्गत एक जाति आनुवंशिक स्तर पर अपने वितरण क्षेत्र में बहुत विविधता दर्शा सकती है। भारत 50 हज़ार से अधिक आनुवंशिक रूप भिन्न धान की तथा 1,000 से अधिक आम की जातियाँ हैं ।
♦ जातीय ( स्पीशीज) विविधता : यह भिन्नता जाति के स्तर पर है, जैसे- भारत में पश्चिमी घाट की उभयचर जातियों की विविधता पूर्वी घाट से अधिक है।
♦ पारिस्थितिकीय विविधता : यह विविधता पारितंत्र स्तर पर होती है, जैसे- भारत में स्थित रेगिस्तान, वर्षा वन, मैंग्रोव प्रवाल भित्ति तथा एल्पाइन वन आदि ।
2. आई. यू. सी. एन. द्वारा लुप्त जातियों (स्पीशीज) को किस सूची के अंतर्गत रखा जाता है?
(a) काली सूची
(b) हरी सूची
(c) लाल सूची
(d) नीली सूची
उत्तर : (c)
व्याख्या : आई.यू.सी.एन. द्वारा लुप्त जातियों (स्पीशीज) को लाल सूची के अंतर्गत रखा जाता है।
3. पृथ्वी पर उपस्थित संपूर्ण जंतुओं में सर्वाधिक संख्या है-
(a) कीटों की
(b) क्रस्टेशिया की
(c) मोलस्का की
(d) पक्षियों की
उत्तर : (a)
व्याख्या: पृथ्वी पर आकलित जातियों में से 70 प्रतिशत से अधिक जंतु हैं, जबकि शैवाल, कवक, ब्रायोफाइट, आवृत्तबीजी तथा अनावृत्तबीजियों जैसे पादप 22 प्रतिशत से अधिक नहीं हैं। जंतुओं में कीट सबसे अधिक समृद्ध जातीय वर्ग समूह है, जो संपूर्ण जातियों के 70 प्रतिशत से अधिक. है। इसका अर्थ यह है कि इस ग्रह में प्रत्येक 10 जंतुओं में 7 कीट हैं।
4. पादपों एवं प्राणियों की अधिकतम विविधता पाई जाती है-
(a) शंकुधारी वनों में
(b) उष्णकटिबंधीय वर्षा वनों में
(c) शीतोष्ण कटिबंधीय वनों में
(d) मरुस्थलीय क्षेत्रों में
उत्तर : (b)
व्याख्याः सामान्यतः पृथ्वी पर भूमध्य रेखा से ध्रुवों की ओर जाने पर पादपों एवं जंतुओं की विविधता घटती जाती है। दक्षिणी अमेरिका के अमेजन उष्णकटिबंधीय वर्षा वनों की जैव विविधता पृथ्वी पर सबसे अधिक है।
5. पृथ्वी पर अन्य क्षेत्रों की अपेक्षा उष्णकटिबंधीय क्षेत्र में सर्वाधिक जैव विविधता पाए जाने का/के प्रमुख कारण है/हैं-
1. इस क्षेत्र में हिमनदन ( ग्लेशिएशन) का अपेक्षाकृत अबाधित रहना ।
2. निम्न मौसमी परिवर्तन ।
3. इस क्षेत्र को अधिक सौर ऊर्जा की प्राप्ति ।
नीचे दिये गए कूट का प्रयोग कर सही उत्तर चुनिये ।
(a) केवल 1
(b) केवल 2 और 3
(c) 1, 2 और 3
(d) केवल 3
उत्तर : (c)
व्याख्या : उपर्युक्त सभी कथन सत्य हैं।
उष्णकटिबंधीय क्षेत्र में पृथ्वी की सर्वाधिक जैव विविधता पाई जाती है। पारिस्थितिक तथा जैव विकासविदों ने बहुत सी परिकल्पनाएँ दी हैं, जिनमें से कुछ मुख्य निम्नलिखित हैं-
• जाति उद्भवन (स्पीशियन) लंबे समय का कार्य है । शीतोष्ण क्षेत्र में प्राचीन समय से बार-बार हिमनदन (ग्लेशिएशन) होता रहा है, जबकि उष्णकटिबंधीय क्षेत्र लाखों वर्षों से अपेक्षाकृत अबाधित रहा है। इसी. कारण जाति विकास तथा विविधता के लिये बहुत समय मिला ।
• उष्णकटिबंध पर्यावरण, शीतोष्ण पर्यावरण से भिन्न तथा कम मौसमीय परिवर्तन दर्शाता है। यह स्थिर पर्यावरण निकेत विशिष्टीकरण (निकस्पेशिएलाइजेशन) को प्रोत्साहित करता रहा है, जिनकी वजह से अधिकाधिक जाति विविधता हुई।
• उष्णकटिबंधीय क्षेत्रों को अधिक सौर ऊर्जा उपलब्ध होती है, जिससे उत्पादन अधिक होता है। इससे परोक्ष रूप से जैव विविधता अधिक होती है।
6. निम्नलिखित में से किस क्षेत्र को पृथ्वी का फेफड़ा कहा जाता है?
(a) ऑस्ट्रेलिया के उष्णकटिबंधीय वर्षावनों को ।
(b) भारत के पश्चिमी घाट को |
(c) अमेजन वर्षा – वन को ।
(d) उत्तरी अमेरिका के शीतोष्ण कटिबंधीय क्षेत्र को ।
उत्तर : (c)
व्याख्या: अमेजन वर्षा वन को ‘पृथ्वी का फेफड़ा’ कहा जाता है। इस विशाल वन में पौधों एवं जंतुओं की करोड़ों जातियाँ (स्पीशीज) निवास करती हैं।
7. निम्नलिखित में से कौन-सा जैव विविधता के ह्रास का सर्वाधिक प्रमुख कारण है?
(a) ज्वालामुखी क्रियाएँ
(b) विदेशज प्रजातियों का आक्रमण
(c) सहविलुप्तता
(d) आवासीय क्षति तथा विखंडन
उत्तर : (d)
व्याख्या: जंतुओं एवं पौधों के विलुप्तीकरण का सबसे प्रमुख कारण आवासीय क्षति तथा विखंडन है। उष्णकटिबंधीय वर्षा वनों में होने वाली आवासीय क्षति इसका सबसे अच्छा उदाहरण है। एक समय वर्षा वन पृथ्वी के 14 प्रतिशत क्षेत्र में फैले थे लेकिन अब 6 प्रतिशत से अधिक क्षेत्र में नहीं हैं।
8. किसी भी भौगोलिक क्षेत्र में जैव विविधता की क्षति के लिये ज़िम्मेदार हो सकते हैं-
1. आवासीय क्षति तथा विखंडन
2. अतिदोहन
3. सहविलुप्तता
4. शाकाहार को बढ़ावा देना
नीचे दिये कूट का प्रयोग कर सही उत्तर चुनिये-
(a) केवल 1 और 2
(b) केवल 1, 2 और 3
(c) केवल 3 और 4
(d) केवल 2, 3 और 4
उत्तर : (b)
व्याख्या : जातीय विलोपन की बढ़ती हुई क्षति मुख्य रूप से मानवीय क्रियाकलापों के कारण है। इसके चार मुख्य कारण हैं-
1. आवासीय क्षति तथा विखंडन
2. अतिदोहन
3. विदेशीय जातियों का आक्रमण
4. सहविलुप्तता
9. जैव विविधता के ‘हॉट-स्पॉट’ क्षेत्र के संबंध में निम्नलिखित कथनों पर विचार कीजिये-
1. यहाँ पर जातीय समृद्धि बहुत अधिक होती है।
2. ये क्षेत्र केवल उष्णकटिबंधीय क्षेत्रों में ही पाए जाते हैं।
3. इन क्षेत्रों को जैव विविधता संरक्षण के स्वस्थाने (इन सिटू) संरक्षण के अंतर्गत शामिल किया जाता है।
उपर्युक्त में से कौन सा/से कथन सत्य है/हैं?
(a) केवल 1 और 2
(b) केवल 2
(c) केवल 1 और 3
(d) केवल 2 और 3
उत्तर : (c)
व्याख्या: केवल पहला व तीसरा कथन सत्य है।
• जैव विविधता के ‘हॉट स्पॉट’ वे क्षेत्र होते हैं, जहाँ पर जातीय समृद्धि बहुत अधिक और उच्च स्थानिक (एंडेमिना) जातियाँ होती हैं, जो अन्य स्थानों पर नहीं होती हैं। यद्यपि संसार के सारे हॉट स्पॉट आपस मिलकर पृथ्वी के दो प्रतिशत से भी कम हैं।
• हॉट स्पॉट क्षेत्र मुख्यतः उष्णकटिबंधीय क्षेत्रों में पाए जाते हैं। ये क्षेत्र उष्णकटिबंधीय से बाहर भी विस्तृत हैं।
• ‘हॉट स्पॉट’ क्षेत्रों का संरक्षण स्वस्थाने (इनसिटू) सरक्षण के अंतर्गत आता है।
10. निम्नलिखित में से कौन सा/से ‘हॉट स्पॉट’ क्षेत्र भारत में विस्तृत है/है?
1. पश्चिमी घाट और श्रीलंका
2. हिमालय
3. इंडो वर्मा
4. सुंडालैंड
नीचे दिये गए कूट का प्रयोग कर सही उत्तर चुनिये-
(a) केवल 1 और 2
(b) केवल 1. 3 और 4
(c) केवल 2 और 4
(d) उपर्युक्त सभी
उत्तर : (d)
व्याख्या: भारत में जैव विविधता के उपर्युक्त चारों ‘हॉट स्पॉट’ क्षेत्र विस्तृत हैं।
11. निम्नलिखित में से कौन-से जैव विविधता संरक्षण के स्वस्थाने ( इन सिद्ध) संरक्षण पद्धति के अंतर्गत शामिल हैं?
1. राष्ट्रीय उद्यान
2. वनस्पति उद्यान
3. जैवमंडल आरक्षित क्षेत्र (बायोस्फीयर रिज़र्व )
4. वन्यजीव अभयारण्य
कूट :
(a) केवल 1 और 2
(b) केवल 2, 3 और 4
(c) केवल 1 3 और 4
(d) उपर्युक्त सभी
उत्तर : (c)
व्याख्या : जैव विविधता संरक्षण के स्वस्थाने (इन सिटू) संरक्षण पद्धति के अंतर्गत राष्ट्रीय उद्यान, वन्यजीव अभयारण्य, जैवमंडल आरक्षित क्षेत्र (बायोस्फीयर रिज़र्व) आते हैं।
12. जैव विविधता संरक्षण के बाह्यस्थाने (एक्स सीटू) संरक्षण विधि के संबंध में नीचे दिये गए कथनों पर विचार कीजिये-
1. इस पद्धति के अंतर्गत संकटापन्न पादप एवं जंतुओं को उनके प्राकृतिक आवास में ही संरक्षित किया जाता है।
2. जंतु उद्यान, वनस्पति उद्यान एवं जीव सफारी का निर्माण भी इसी के अंतर्गत किया जाता है।
उपर्युक्त में से कौन सा/ये कथन सत्य है/है?
(1) केवल 1
(b) केवल 2
(c) 1 और 2 दोनों
(d) न तो 1 और न ही 2
उत्तर : (b)
व्याख्या: पहला कथन असत्य है। संकटापन्न पाइप एवं जंतुओं को उनके प्राकृतिक आवास में संरक्षित स्वस्थाने (इन सिद्ध) संरक्षण विधि से किया जाता है। बाह्यस्थाने (एक्स सीटू) संरक्षण विधि में संकटापन्न पादपों एवं जंतुओं की उनके प्राकृतिक आवास से अलग एक विशेष स्थान पर उनकी अच्छी देखभाल की जाती है और इन्हें सावधानीपूर्वक संरक्षित किया जाता है। दूसरा कथन सत्य है। जंतू उद्यान, वनस्पति उद्यान तथा जीव सफारी का निर्माण बाह्यस्थाने संरक्षण पद्धति के अंतर्गत किया गया है। ऐसे बहुत से जंतु, हैं जो कि बनों से विलुप्त हो गए हैं, लेकिन जंतु उद्यानों में सुरक्षित हैं।

16. पर्यावरण के मुद्दे

1. निम्नलिखित कथनों पर विचार कीजिये-
1. स्थिर विद्युत अवक्षेपित्र (इलेक्ट्रोस्टैटिक प्रेसिपेटेटर) कणिकीय पदार्थों को हटाने का सर्वोत्तम तरीका है।
2. पीएम 4.5 आकार के कणिकीय पदार्थ मानव स्वास्थ्य के लिये सबसे अधिक नुकसानदेह हैं।
उपर्युक्त कथनों में से कौन सा/से सही है/हैं?
(a) केवल 1
(b) केवल 2
(c) 1 और 2 दोनों
(d) न तो 1 और न ही 2
उत्तर : (a)
व्याख्या: पहला कथन सत्य है। कणिकीय पदार्थों की निकालने के लिये सबसे अधिक व्यापक तरीका स्थिर विद्युत अवक्षेपित्र (इलैक्ट्रोस्टैटिक प्रेसिपेटेटर) है जो ताप विद्युत संयंत्र के निर्वातक (Exhaust ) में मौजूद 99 प्रतिशत कणिकीय पदार्थों को हटा देता है। स्थिर विद्युत अवक्षेपित्र में एक इलेक्ट्रोड तार होता है, जिससे होकर हज़ारों वोल्ट गुज़रता है तथा यह करीना उत्पन्न करता है और इससे इलेक्ट्रॉन निकलते हैं। ये इलेक्ट्रॉन
• धूल के कणों से सट जाते हैं और इन्हें ऋण आवेश प्रदान करते हैं। इससे संचायक पट्टिकाएँ नीचे की ओर आ जाती हैं और आवेशित धूल कणों को आकर्षित करती हैं।
• दूसरा कथन असत्य है। केंद्रीय प्रदूषण नियंत्रण बोर्ड (CPCB) के अनुसार 2.5 माइक्रोमीटर या कम व्यास के आकार (पीएम 2.5) के कणिकीय पदार्थ मानव स्वास्थ्य के लिये सबसे अधिक नुकसानदेह हैं। जब मनुष्य साँस लेता है तो ये सूक्ष्म कणिकीय पदार्थ फेफड़ों के भीतर चले जाते हैं तथा ये फेफड़ों को क्षति पहुँचाते हैं।
2. निम्नलिखित युग्मों पर विचार कीजिये –
1. पर्यावरण (संरक्षा) अधिनियम – 1986
2. जल प्रदूषण निरोध एवं नियंत्रण अधिनियम – 1974
3. वायु प्रदूषण निरोध एवं नियंत्रण अधिनियम – 1981
उपर्युक्त में से कौन-सा/से युग्म सही सुमेलित है/हैं?
(a) केवल 1
(b) केवल 2 और 3
(c) केवल 3
(d) उपर्युक्त तीनों
उत्तर : (d)
व्याख्या: उपर्युक्त तीनों युग्म सुमेलित हैं।
3. डीजल या पेट्रोल की जगह संपीड़ित प्राकृतिक गैस (सीएनजी) को प्रयोग करने का प्रमुख कारण है-
1. वाहनों में सीएनजी के जलने के बाद अवशेष कम बचता है।
2. इसमें किसी तरह की मिलावट नहीं की जा सकती है।
3. सीएनजी गैस का पेट्रोल/डीजल से सस्ता होना ।
नीचे दिये गए कूट का प्रयोग कर सही उत्तर चुनिये-
(a) केवल 1
(b) केवल 2 और 3
(c) केवल 1, 2 और 3
(d) केवल 3
उत्तर : (c)
व्याख्या : डीजल या पेट्रोल की जगह संपीड़ित प्राकृतिक गैस (सीएनजी) प्रयोग करने के प्रमुख कारण हैं-
• वाहनों में सीएनजी सबसे अच्छी तरह से जलता है और बहुत ही कम मात्रा में जलने से बचता है, जबकि डीजल या पेट्रोल के मामले में ऐसा नहीं है।
• सीएनजी डीजल व पेट्रोल से सस्ता है।
• सीएनजी की चोरी नहीं की जा सकती है और डीजल या पेट्रोल की तरह इसे अपमिश्रित ( मिलावट ) नहीं किया जा सकता।
• इसका मुख्य संघटक मीथेन होती है। इसके अलावा इसमें ईथेन और प्रोपेन गैसें भी होती हैं।
• सीएनजी चूँकि प्राकृतिक गैस का संपीड़ित रूप है। इसलिये संपीड़न से हानिकारक गैसों का उत्सर्जन काफी कम हो जाता है। इस कारण से यह एक पर्यावरण हितैषी ईंधन है।
4. निम्नलिखित कथनों पर विचार कीजिये-
1. जलाशयों में काफी मात्रा में पोषकों की उपस्थिति के कारण प्लवकीय शैवाल की अतिशय वृद्धि को शैवाल प्रस्फुटन कहते हैं।
2. शैवाल प्रस्फुटन से जल की गुणवत्ता पर कोई प्रभाव नहीं पड़ता है।
उपर्युक्त कथनों में से कौन सा/से सही है/हैं?
(a) केवल 1
(b) केवल 2
(c) 1 और 2 दोनों
(d) न तो । और न ही 2
उत्तर : (a)
व्याख्या: पहला कथन सत्य है। जब जलाशयों में काफी मात्रा में पोषकों की उपस्थिति पाई जाती है, तब इन पोषकों के कारण प्लवकीय (मुक्त – प्लावी) शैवाल की अतिशय वृद्धि होती है। इसे शैवाल प्रस्फुटन ( अलगल ब्लूम) कहा जाता है। शैवालों की वृद्धि के कारण जलाशयों का रंग विशेष प्रकार का हो जाता है।
• दूसरा कथन असत्य है। शैवाल प्रस्फुटन के कारण जल की गुणवत्ता घट जाती है और जलीय पारितंत्र पर गंभीर दुष्प्रभाव पड़ता है। कुछ प्रस्फुटनकारी शैवाल मनुष्य और जानवरों के लिये अतिशय विषैले होते हैं।
5. किस पादप को बंगाल का आतंक कहा जाता है?
(a) नीलाशोण
(b) हायसिंध (आइकोर्निया केसिपीज )
(c) सारगासम
(d) इनमें से कोई नहीं।
उत्तर : (b)
व्याख्या : हायसिंध (Eichhornia Crassipes) जल पादप हैं। जो विश्व के सबसे अधिक समस्या उत्पन्न करने वाले जलीय खरपतवार हैं इसे बंगाल का आतंक कहा जाता है। ये पादप सुपोषी जलाशयों में काफी वृद्धि करते हैं तथा पारितंत्र गति को असंतुलित कर देते हैं।
6. निम्नलिखित में से कौन-सा कथन जैव आवर्धन (बायोमैग्निफिकेशन) को सही तरीके से परिभाषित करता है?
(a) खाद्य श्रृंखला में नए जीवों का समावेश ।
(b) किसी जलीय पारितंत्र में विदेशज प्रजातियों का आक्रमण।
(c) खाद्य श्रृंखला में क्रमिक पोषण स्तर ( ट्रॉफिक लेवल) पर आविषाक्त की सांद्रता में वृद्धि होना ।
(d) जीवों द्वारा पर्यावरण से विषैले पदार्थों का निष्कासन ।
उत्तर : (c)
व्याख्या: जैव आवर्धन (बायोमैग्निफिकेशन) का तात्पर्य है, क्रमिक पोषण स्तर ( ट्रॉफिक लेवल) पर आविषाक्त की सांद्रता में वृद्धि होना। इसका कारण है जीव द्वारा सगृहीत अविषालु पदार्थ उपापचयित या उत्सर्जित नहीं हो सकता और इस प्रकार यह अगले उच्चतर पोषण स्तर पर पहुँच जाता है। उद्योगों के अपशिष्ट जल में विद्यमान प्राय: कुछ विषैले पदार्थों में जलीय खाद्य श्रृंखला जैव आवर्धन कर सकते हैं।
7. किसी भी झील की अधिक उम्र होने के साथ उसमें पोषक तत्त्वों के बढ़ने से जैव समृद्धि बढ़ती जाती है। यह परिघटना कहलाती है-
(a) इकोलॉजिकल फुटप्रिंटिंग
(b) रासायनिक चक्र
(c) सुपोषण
(d) जैव उपचारण
उत्तर : (c)
व्याख्या : सुपोषण (Eutrophication) झील का प्राकृतिक काल प्रभावन (एजिंग) को दर्शाता है यानी झील अधिक उम्र की हो जाती है। यह इसके जल की जैव समृद्धि के कारण होता है। तरुण ( कम उम्र की ) झील का जल शीतल और स्वच्छ होता है। समय के साथ-साथ, इसमें सरिता के जल के साथ पोषक तत्त्व, जैसे- नाइट्रोजन और फॉस्फोरस आते रहते हैं जिसके कारण जलीय जीवों में वृद्धि होती रहती है। जैसे-जैसे झील की उर्वरता बढ़ती है, वैसे-वैसे पादप और प्राणी जीवन बढ़ने लगता है और कार्बनिक अवशेष झील के तल में बैठने लगते हैं। सैकड़ों वर्षों में इसमें जैसे-जैसे अवसाद (सिल्ट) और जैव मलबे (आर्गेनिक मलबा ) का ढेर लगता जाता है वैसे-वैसे झील उथली और गर्म होती जाती है। झील में ठंडे पर्यावरण वाले जीवों स्थान पर उष्णजल जीव रहने लगते हैं। कच्छ पादप उथली जगह पर जड़ जमा लेते हैं और झील की मूल द्रोणी (बेसिन) को भरने लगते हैं। उथले झील में अब कच्छ पादप उग आते हैं और मूल झील बेसिन उनसे भर जाता है। कालांतर में झील काफी संख्या में प्लावी पादपों ( दलदल / बॉग) से भर जाता है और अंत में यह भूमि में परिवर्तित जाती है। जलवायु, झील का साइज और अन्य कारकों के अनुसार झील का यह प्राकृतिक काल-प्रभावन हजारों वर्षों में होता है। फिर भी मनुष्य के क्रिया-कलाप, जैसे उद्योगों और घरों के बहि:स्राव काल – प्रभावन प्रक्रम में मूलतः तेज़ी ला सकते हैं। इस प्रक्रिया को संवर्ध (कल्चरल) या त्वरित सुपोषण (एक्सिलरेटेड यूट्रॉफिकेशन) कहा जाता है।
8. झील में शैवाल की अतिशय वृद्धि के कारण अरमणीक मलफेन (स्कम) बनते हैं तथा झील से अरुचिकर गंध निकलने लगती है। इसका प्रमुख कारण है-
1. जलाशय में नाइट्रेट और फॉस्फोरस जैसे प्रदूषकों की अत्यधिक वृद्धि ।
2. जलीय पारितंत्र में विदेशज प्रजातियों का आक्रमण ।
3. वातावरण में सल्फर डाइऑक्साइड गैस की मात्रा में वृद्धि होना ।
कूट :
(a) केवल 1
(b) केवल 2 और 3
(c) केवल 1 और 3
(d) केवल 1, 2 और 3
उत्तर : (a)
व्याख्या : झीलों का वाहित मल, कृषि और औद्योगिक अपशिष्ट के कारण तीव्र सुपोषण हुआ है। इसके मुख्य संदूषक नाइट्रेट और फॉस्फोरस हैं, जो पौधों के लिये पोषक का कार्य करते हैं। इन पोषकों के कारण शैवाल की वृद्धि अति उद्दीप्त होती है, जिसकी वजह से अरमणीक मलफेन (स्कम) बनते हैं तथा अरुचिकर गंध निकलती है। ऐसा होने से जल में विलीन ऑक्सीजन जो अन्य जल- जीवों के लिये अनिवार्य (वाइटल ) है, समाप्त हो जाती है। साथ ही झील में बहकर आने वाले अन्य प्रदूषक संपूर्ण मत्स्य समष्टि को विषाक्त कर सकता है। जिनके अपघटन के अवशेष से जल में विलीन ऑक्सीजन की मात्रा और कम हो जाती है। इस प्रकार झील घुट-घुट कर मर सकती है।
9. निम्नलिखित में से कौन-सी बीमारियाँ प्रदूषित जल के उपयोग से होती हैं?
1. पेचिश (अतिसार)
2. टाइफाइड
3. पीलिया (जांडिस )
4. हैजा (कॉलरा)
कूट :
(a) केवल 1 और 2
(b) केवल 1, 2 और 4
(c) केवल 1 और 3
(d) उपर्युक्त सभी।
उत्तर : (d)
व्याख्या : हमारे घरों के साथ-साथ अस्पतालों के वाहित मल में बहुत से अवांछित रोगजनक सूक्ष्मजीव हो सकते हैं और उचित उपचार के बिना | इसको जल में विसर्जित करने से मानव को कठिन रोग जैसे- पेचिस (अतिसार), टाइफाइड, पीलिया (जांडिस), हैजा (कॉलरा) आदि हो सकते हैं।
10. वायुमंडल में उपस्थित ग्रीन हॉउस गैसों में सर्वाधिक उपस्थित गैसों का उनके प्रतिशत के आधार पर सही क्रम है- 
(a) कार्बन डाइऑक्साइड – मीथेन – क्लोरो फ्लोरोकार्बन
(b) कार्बन डाइऑक्साड – मीथेन – सल्फर डाइऑक्साइड
(c) मीथेन – कार्बन डाइऑक्साइड – नाइट्रस ऑक्साइड
(d) मीथेन – क्लोरो फ्लोरोकार्बन – सल्फर डाइऑक्साइड
उत्तर : (a)
व्याख्या: ग्रीन हाउस गैसों में सर्वाधिक योगदान कार्बन डाइऑक्साइड (60%) का है। इसके बाद मीथेन (20%), क्लोरो फ्लोरोकार्बन ( 14%) तथा नाइट्रस ऑक्साइड (6%) का है।
11. निम्नलिखित कथनों पर विचार कीजिये-
1. ग्रीन हाउस प्रभाव के कारण पृथ्वी की सतह और वायुमंडल दोनों गर्म हो जाते हैं।
2. ग्रीन हाउस गैसें पृथ्वी से उत्सर्जित दीर्घ तरंग विकिरण को अवशोषित कर पुनः पृथ्वी की ओर उत्सर्जित करती हैं।
उपर्युक्त में से कौन-सा/से कथन सत्य है/हैं?
(a) केवल 1
(b) केवल 2
(c) 1 और 2 दोनों
(d) न तो 1 और न ही 2
उत्तर : (c)
व्याख्या : उपर्युक्त दोनों कथन सत्य हैं।
• ग्रीन हाउस प्रभाव प्राकृतिक रूप से होने वाली परिघटना है, जिसके कारण पृथ्वी की सतह और वायुमंडल गर्म हो जाते हैं। यदि ग्रीन हाउस प्रभाव नहीं होता तो आज पृथ्वी का औसत तापमान 15 डिग्री सेंटीग्रेड रहने की बजाय ठंडा होकर 18 डिग्री सेंटीग्रेड रहता ।
• ग्रीन हाउस गैस पृथ्वी द्वारा उत्सर्जित दीर्घ तरंग (अवरक्त ) विकिरण को अवशोषित करती है और पुनः पृथ्वी की ओर भेज देती है। यह चक्र अनेक बार होता रहता है। इस प्रकार पृथ्वी की सतह और निम्नतर वायुमंडल गर्म होता रहता है।
12. भारत की राष्ट्रीय वन नीति 1988 के अनुसार मैदानी इलाकों और पर्वतीय क्षेत्रों में कितना प्रतिशत वन क्षेत्र होना चाहिये ?
(a) मैदानी – 23, पर्वतीय – 77 प्रतिशत
(b) मैदानी – 33, पर्वतीय – 67 प्रतिशत
(c) मैदानी – 27, पर्वतीय – 73 प्रतिशत
(d) मैदानी – 35, पर्वतीय – 65 प्रतिशत
उत्तर : (b)
व्याख्या : भारत की राष्ट्रीय वन नीति, 1988 के तहत सिफारिश की गई कि मैदानी इलाकों में 33 प्रतिशत वन क्षेत्र तथा पर्वतीय क्षेत्रों में 67 प्रतिशत जंगल क्षेत्र होना चाहिये।
13. ओजोन परत की मोटाई मापने की यूनिट है-
(a) नॉट
(b) डॉब्सन
(c) मैक
(d) पाइरहिलियोमीटर
उत्तर : (b)
व्याख्या : वायुमंडल के निचले भाग से लेकर शिखर तक के वायु स्तंभ (कॉलम) में ओज़ोन की मोटाई डॉब्सन यूनिट में मापी जाती है।
14. हमारी आँख का कौन-सा भाग पराबैंगनी – बी विकिरण का अवशोषण करता है?
(a) रेटिना
(b) स्वच्छमंडल (कॉर्निया)
(c) आइरिस
(d) उपर्युक्त सभी
उत्तर : (b)
व्याख्या: हमारी आँख का स्वच्छमंडल (कॉर्निया) पराबैंगनी – बी (यूवी – बी) विकिरण का अवशोषण करता है। इसकी उच्च मात्रा के कारण कॉर्निया का शोथ हो जाता है। जिसे हिम अंधता मोतियाबिंद आदि कहा जाता है।
15. ओजोन परत के क्षय के कारण पराबैंगनी किरण पृथ्वी पर रहने वाले जीवों पर कौन – सा / से प्रभाव डालेगी / डालेंगी?
1. सजीवों के डीएनए और प्रोटीन को भंग कर देंगी।
2. त्वचा कैंसर।
3. ग्रीन हाउस प्रभाव में वृद्धि ।
कूट:
(a) केवल 1
(b) केवल 1 और 2
(c) केवल 3
(d) 1, 2 और 3
उत्तर : (b)
व्याख्याः समतापमंडल में ओजोन परत का क्षय विस्तृत रूप से होता है। लेकिन यह क्षय अंटार्कटिका क्षेत्र में खासकर विशेषरूप से अधिक होता है। इसके फलस्वरूप यहाँ काफी बड़े क्षेत्र में ओज़ोन की परत काफी पतली हो गई है, जिसे सामान्यतः ओजोन छिद्र ( ओज़ोन होल ) कहा जाता है। पराबैंगनी -बी (यूवी – बी) की अपेक्षा छोटे तरंगदैर्ध्य युक्त पराबैंगनी (यूवी) विकिरण पृथ्वी के वायुमंडल द्वारा लगभग पूरी अवशोषित हो जाती है। बशर्ते ओज़ोन स्तर ज्यों का त्यों रहे ।
सजीवों के डीएनए और प्रोटीन खासकर पराबैंगनी (यूवी – बी) किरणों को अवशोषित करते हैं और इसकी उच्च ऊर्जा इन अणुओं के रासायनिक आबंध (केमिकल बॉण्डस) को भंग कर देती हैं जिसके कारण उत्परिवर्तन भी हो सकता है। इसके कारण त्वचा में बुढ़ापे के लक्षण दिखते हैं, इसकी कोशिकाएँ, क्षतिग्रस्त हो जाती हैं और विविध प्रकार के त्वचा कैंसर हो सकते हैं।
16. ओज़ोन परत को नुकसान पहुँचाने के लिये निम्न में से कौन-सी गैस ज़िम्मेवार हैं?
(a) ऑक्सीजन (O2)
(b) कार्बन डाइऑक्साइड (CO2)
(c) नाइट्रोजन (N2)
(d) क्लोरोफ्लोरोकार्बन (CFC)
उत्तर : (d)
व्याख्या : सर्वप्रथम समतापमंडल में ओजोन के उत्पादन और अवक्षय निम्नीकरण में संतुलन होना चाहिये। लेकिन वर्तमान में, क्लोरो फ्लोरोकार्बन (CFCs) के द्वारा ओज़ोन निम्नीकरण बढ़ जाने से इसका संतुलन बिगड़ गया है। वायुमंडल के निचले भाग में उत्सर्जित CFCs ऊपर की ओर उठता है और यह समतापमंडल में पहुँचता है तथा समतापमंडल में पराबैंगनी किरणें उस पर कार्य करती हैं। समतापमंडल में जो भी क्लोरो फ्लोरोकार्बन जुड़ते जाते हैं, उनका ओज़ोन स्तर पर स्थायी और सतत प्रभाव पड़ता है।

Leave a Reply

Your email address will not be published. Required fields are marked *